Nursing SS #2

Réussis tes devoirs et examens dès maintenant avec Quizwiz!

What is the most likely cause for early decelerations in the fetal heart rate (FHR) pattern? a. Altered fetal cerebral blood flow b. Umbilical cord compression c. Uteroplacental insufficiency d. Spontaneous rupture of membranes

a. Altered fetal cerebral blood flow

Fetal well-being in labor can be measured by the response of the fetal heart rate (FHR) to uterine contractions. Match the characteristic of normal uterine activity during labor with the correct description. a. Frequency b. Duration c. Strength d. Resting tone e. Relaxation time 1. Commonly 45 seconds or more in the second stage of labor 2. Generally ranging from two to five contractions per 10 minutes of labor 3. Average of 10 mm Hg 4. Peaking at 40 to 70 mm Hg in the first stage of labor 5. Remaining fairly stable throughout the first and second stages

1. Commonly 45 seconds or more in the second stage of labor= e. Relaxation time 2. Generally ranging from two to five contractions per 10 minutes of labor= a. Frequency 3. Average of 10 mm Hg= d. Resting tone 4. Peaking at 40 to 70 mm Hg in the first stage of labor= c. Strength 5. Remaining fairly stable throughout the first and second stages= b. Duration

Herbal preparations have long been used for the management of menstrual problems, including dysmenorrhea, cramping and discomfort, and breast pain. For the nurse to counsel adequately the client who elects to use this alternative modality, understanding the action of these herbal preparations is important. Match the herbal medicine with the appropriate action. a. Uterine antispasmodic b. Uterotonic c. Antiinflammatory d. Estrogen-like luteinizing hormone suppressant e. Decreases prolactin levels 1. Fennel, dong quai 2. Chaste tree fruit 3. Black cohosh root 4. Valerian, wild yam 5. Ginger

1. Fennel, dong quai= b. Uterotonic 2. Chaste tree fruit= e. Decreases prolactin levels 3. Black cohosh root= d. Estrogen-like luteinizing hormone suppressant 4. Valerian, wild yam= a. Uterine antispasmodic 5. Ginger= c. Antiinflammatory

All pregnant women should be instructed to recognize and report potential complications for each trimester of pregnancy. Match the sign or symptom with a possible cause. a. Severe vomiting in early pregnancy b. Epigastric pain in late pregnancy c. Severe backache and flank pain d. Decreased fetal movement e. Glycosuria 1. Fetal jeopardy or intrauterine fetal death 2. Kidney infection or stones 3. Gestational diabetes 4. Hyperemesis gravidarum 5. Hypertension, preeclampsia

1. Fetal jeopardy or intrauterine fetal death= d. Decreased fetal movement 2. Kidney infection or stones= c. Severe backache and flank pain 3. Gestational diabetes= e. Glycosuria 4. Hyperemesis gravidarum= a. Severe vomiting in early pregnancy 5. Hypertension, preeclampsia= b. Epigastric pain in late pregnancy

The physiologic changes that occur during the reversal of the processes of pregnancy are distinctive; however, they are normal. To provide care during this recovery period, the nurse must synthesize knowledge regarding anticipated maternal changes and deviations from normal. Match the vital signs finding that the postpartum nurse may encounter with its probable cause. a. Elevated temperature within the first 24 hours b. Rapid pulse c. Elevated temperature at 36 hours postpartum d. Hypertension e. Hypoventilation 1. Puerperal sepsis 2. Unusually high epidural or spinal block 3. Dehydrating effects of labor 4. Hypovolemia resulting from hemorrhage 5. Excessive use of oxytocin

1. Puerperal sepsis = c. elevated temp at 36 hrs 2. Unusually high epidural or spinal block= e. Hypoventilation 3. Dehydrating effects of labor= a. Elevated temperature within the first 24 hours 4. Hypovolemia resulting from hemorrhage= b. Rapid pulse 5. Excessive use of oxytocin= d. Hypertension

The vaginal examination is an essential component of labor assessment. It reveals whether the client is in true labor and enables the examiner to determine whether membranes have ruptured. The vaginal examination is often stressful and uncomfortable for the client and should be performed only when indicated. Match the correct step number, from 1 to 7, with each component of a vaginal examination of the laboring woman. a. After obtaining permission, gently insert the index and middle fingers into the vagina. b. Explain the findings to the client. c. Position the woman to prevent supine hypotension. d. Use sterile gloves and soluble gel for lubrication. e. Document findings and report to the health care provider. f. Cleanse the perineum and vulva, if necessary. g. Determine dilation, presenting part, status of membranes, and characteristics of amniotic fluid. 1. Step 1 2. Step 2 3. Step 3 4. Step 4 5. Step 5 6. Step 6 7. Step 7

1. Step 1= d. Use sterile gloves and soluble gel for lubrication. 2. Step 2= c. Position the woman to prevent supine hypotension. 3. Step 3= f. Cleanse the perineum and vulva, if necessary. 4. Step 4= a. After obtaining permission, gently insert the index and middle fingers into the vagina. 5. Step 5= g. Determine dilation, presenting part, status of membranes, and characteristics of amniotic fluid. 6. Step 6= b. Explain the findings to the client. 7. Step 7= e. Document findings and report to the health care provider.

The female athlete triad includes which common menstrual disorder? a. Amenorrhea b. Dysmenorrhea c. Menorrhagia d. Metrorrhagia

a. Amenorrhea

Which nursing intervention would result in an increase in maternal cardiac output? a. Change in position b. Oxytocin administration c. Regional anesthesia d. IV analgesic

a. Change in position

Which component of the physical examination are Leopold's maneuvers unable to determine? a. Gender of the fetus b. Presenting fetal part c. Fetal lie d. Estimated fetal size

a. Gender of the fetus

The clinic nurse is evaluating a patient with a vaginal infection. The nurse knows that the normal vaginal pH is in which range?

4.0 to 5.0

The clinic nurse is evaluating an adolescent with menses that have stopped occurring. The nurse understands that which minimum amount of time should the menses be absent after a period of menstruation to be diagnosed as secondary amenorrhea?

6 months

After a treatment plan for acne has been initiated, which time period should the nurse explain to an adolescent before improvement will be seen?

6 to 8weeks

Which alternative approaches to relaxation have proven successful when working with the client in labor? (Select all that apply.) a. Aromatherapy b. Massage c. Hypnosis d. Cesarean birth e. Biofeedback

A. Aromatherapy b. Massage c. Hypnosis e. Biofeedback

A patient is admitted to the intensive care unit for congestive heart failure. Using the situation-background-assessment-recommendation (SBAR) format, put the following statements in the order in which the nurse should report changes to the health care team. a. Our patient was admitted 2 days ago with heart failure and has been receiving furosemide (Lasix) for diuresis, but his urine output has been low. b. I think that our patient needs to be evaluated immediately and may need intubation and mechanical ventilation. c. This is the nurse on the surgical unit. I am calling about our patient in room 3. After assessing him, I am very concerned about his shortness of breath. d. Today, our patient has crackles audible throughout the posterior chest and his O2 saturation is 89%. His condition is very unstable.

ANS: 1. C. This is the nurse on the surgical unit. I am calling about our patient in room 3. After assessing him, I am very concerned about his shortness of breath. 2. A. This is the nurse on the surgical unit. I am calling about our patient in room 3. After assessing him, I am very concerned about his shortness of breath. 3. D. Today, our patient has crackles audible throughout the posterior chest and his O2 saturation is 89%. His condition is very unstable. 4. B. I think that our patient needs to be evaluated immediately and may need intubation and mechanical ventilation. The order of the nurse's statements follows the SBAR format. The nurse explains the situation by identifying herself, then gives the immediate background of the patient. She then assesses the patient and gives her recommendation.

In caring for the woman with disseminated intravascular coagulation (DIC), which order should the nurse anticipate? a. Administration of blood b. Preparation of the client for invasive hemodynamic monitoring c. Restriction of intravascular fluids d. Administration of steroids

a. Administration of blood

A health care provider prescribes danazol (Danocrine), PO, 200 mg, once a day, for an adolescent with endometriosis. The medication label states: Danazol (Danocrine), 100 mg/1 tablet. The nurse prepares to administer one dose. How many tab(s) should the nurse prepare to administer one dose? Fill in the blank. Record your answer as a whole number.

ANS: 2 Follow the formula for dosage calculation. Desired Quantity = Tablets per dose Available 200 mg 1 = 2 tabs 100 mg

A client who had a hysterectomy has a 200-mg dose of ciprofloxacin (Cipro) ordered to infuse in 30 minutes. At what rate should the nurse infuse the medication if the pharmacy provides 200 mg in a 100-mL bag of normal saline? (Record your answer using a whole number.) ___ mL/hr

ANS: 200 mL/hr 100 mL 2 = 200 mL/hr.

Which women should undergo prenatal testing for the human immunodeficiency virus (HIV)? a. All women, regardless of risk factors b. Women who have had more than one sexual partner c. Women who have had a sexually transmitted infection (STI) d. Woman who are monogamous with one partner

a. All women, regardless of risk factors

A woman arrives at the clinic for a pregnancy test. The first day of her LMP was September 10, 2014. Her expected date of birth (EDB) is __________.

ANS: June 17, 2015 Using the Nägele's rule, June 17, 2015, is the correct EDB. The EDB is calculated by subtracting 3 months from the first day of the LMP and adding 7 days + 1 year to the day of the LMP. Therefore, with an LMP of September 10, 2014: September 10, 2014 - 3 months = June 10, 2014 + 7 days = June 17, 2014 + 1 year = June 17, 2015.

A health care provider prescribes leuprolide (Lupron), 3.75 mg, IM, monthly, for a patient with endometriosis. The medication label states: Leuprolide (Lupron) 5 mg/1 ml. The nurse prepares to administer the monthly dose. How many milliliters will the nurse prepare to administer the dose? Fill in the blank. Record your answer using two decimal places.

ANS:0.75 Follow the formula for dosage calculation. Desired Volume = ml per dose Available 75 mg 1 ml = 0.75 ml 5 mg

A health care provider prescribes methylphenidate hydrochloride (Ritalin), PO, 8 mg, twice a day, for an adolescent with bulimia nervosa. The medication label states: Methylphenidate hydrochloride (Ritalin), 4 mg/1 tablet. The nurse prepares to administer one dose. How many tab(s) should the nurse prepare to administer one dose? Fill in the blank. Record your answer as a whole number.

ANS:2 Follow the formula for dosage calculation. Desired Quantity = Tablets per dose Available 8 mg 1 = 2 tabs 4 mg

What menstrual disorders are indications for a pelvic examination? (Select all that apply.) Amenorrhea Dyspareunia Impaired fertility Irregular uterine or vaginal bleeding Dysmenorrhea unresponsive to therapy

Amenorrhea Irregular uterine or vaginal bleeding Dysmenorrhea unresponsive to therapy

What statement regarding chlamydial infections is correct?

Both men and women may have asymptomatic infections.

It is important that women with anogenital warts caused by the human papillomavirus (HPV) receive adequate treatment because this sexually transmitted infection increases the risk of what?

Cervical cancer

The nurse is teaching an adolescent girl strategies to relieve dysmenorrhea. What should the nurse include in the teaching session? (Select all that apply.) Effleurage Diet high in fat Limiting exercise Use of a heating pad Massaging the lower back

Effleurage Use of a heating pad Massaging the lower back

The nurse is teaching an adolescent female about the symptoms of premenstrual syndrome (PMS). What symptoms should the nurse include in the teaching session? (Select all that apply.) Headaches Fluid retention Increased energy Emotional changes Premenstrual cravings

Headaches Fluid retention Emotional changes Premenstrual cravings

The nurse at the family planning clinic conducts a male history for infertility evaluation. Which finding has the greatest implication for this patient's care? a. Practice of nightly masturbation b. Primary anovulation c. High testosterone levels d. Impotence due to alcohol ingestion

Impotence due to alcohol ingestion

What are risk factors of testicular cancer? (Select all that apply.) Hispanic Infertility Alcohol use Tobacco use Family history

Infertility Tobacco use Family history

The nurse is preparing to administer danazol (Danocrine) to a patient with endometriosis. What are the side effects of this medication? (Select all that apply.) Insomnia Hot flashes Amenorrhea Increased libido V aginal secretions

Insomnia Hot flashes Amenorrhea

What is true about pelvic inflammatory disease (PID)?

It may have devastating effects on the reproductive tract of affected adolescents.

The nurse is teaching an adolescent with premenstrual syndrome (PMS) dietary measures to relieve the symptoms of PMS. What should the nurse include in the teaching session? (Select all that apply.) Limit salt in the diet. Limit legumes in the diet. Include red meat in the diet. Include whole grains in the diet. Limit consumption of refined sugar.

Limit salt in the diet. Include whole grains in the diet. Limit consumption of refined sugar.

An adolescent patient has been diagnosed with a vulvovaginal candidiasis (yeast infection). The nurse expects the health care provider to recommend which vaginal cream?

Miconazole (Monistat)

A female college student is planning to become sexually active. She is considering birth control options and desires a method in which ovulation will be prevented. To prevent ovulation while reaching 99% effectiveness in preventing pregnancy, which option should be given the strongest consideration? a. Intrauterine device b. Coitus interruptus c. Natural family planning d. Oral contraceptive pills

Oral contraceptive pills

The nurse is caring for an adolescent male with gynecomastia. What groups of drugs can induce gynecomastia in male adolescents? (Select all that apply.) Oral antibiotics Oral ketoconazoles Calcium channel blockers Histamine-2 receptor blockers Cancer chemotherapeutic agents

Oral ketoconazoles Calcium channel blockers Histamine-2 receptor blockers Cancer chemotherapeutic agents

The nurse is teaching an adolescent female with primary dysmenorrhea foods that are natural diuretics. What foods should the nurse include in the teaching plan? (Select all that apply.) Peaches Asparagus Watermelon Wheat bread Dairy products

Peaches Asparagus Watermelon

The school nurse suspects a testicular torsion in a young adolescent student. What action should the nurse take?

Refer the adolescent for immediate medical evaluation.

What statement is true about gonorrhea?

Treatment of all sexual contacts is essential.

The school nurse is teaching a group of adolescent females which measures to take to prevent genital tract infections. What should the nurse include in the teaching session? (Select all that apply.) Use condoms. Douche once a week. Avoid tight-fitting clothing. Limit exposure to bubble baths. Avoid colored and scented toilet tissue.

Use condoms. Avoid tight-fitting clothing. Limit exposure to bubble baths. Avoid colored and scented toilet tissue.

A sexually active adolescent asks the school nurse about prevention of sexually transmitted infections (STIs). What should the nurse recommend?

Use of condoms

A nurse is conducting a therapeutic session with a patient in the inpatient psychiatric facility. Which remark by the nurse would be an appropriate way to begin an interview session? a. "How shall we start today?" b. "Shall we talk about losing your privileges yesterday?" c. "Let's get started discussing your marital relationship." d. "What happened when your family visited yesterday?"

a. "How shall we start today?"

Which symptom described by a client is characteristic of premenstrual syndrome (PMS)? a. "I feel irritable and moody a week before my period is supposed to start." b. "I have lower abdominal pain beginning on the third day of my menstrual period." c. "I have nausea and headaches after my period starts, and they last 2 to 3 days." d. "I have abdominal bloating and breast pain after a couple days of my period."

a. "I feel irritable and moody a week before my period is supposed to start."

During a one-on-one interaction with the nurse, a patient frequently looks nervously at the door. Select the best comment by the nurse regarding this nonverbal communication. a. "I notice you keep looking toward the door." b. "This is our time together. No one is going to interrupt us." c. "It looks as if you are eager to end our discussion for today." d. "If you are uncomfortable in this room, we can move someplace else."

a. "I notice you keep looking toward the door."

A patient who has been diagnosed with depression is scheduled for cognitive therapy in addition to receiving prescribed antidepressant medication. The nurse understands that the goal of cognitive therapy will be met when what is reported by the patient? a. "I will tell myself that I am a good person when things don't go well at work." b. "My medications will make my problems go away." c. "My family will help take care of my children while I am in the hospital." d. "This therapy will improve my response to neurotransmitter impulses."

a. "I will tell myself that I am a good person when things don't go well at work."

A nurse interacts with patients diagnosed with various mental illnesses. Which statements reflect use of therapeutic communication? (Select all that apply.) a. "Tell me more about that situation." b. "Let's talk about something else." c. "I notice you are pacing a lot." d. "I'll stay with you a while." e. "Why did you do that?"

a. "Tell me more about that situation." c. "I notice you are pacing a lot." d. "I'll stay with you a while."

A patient with acute depression states, "God is punishing me for my past sins." What is the nurse's most therapeutic response? a. "You sound very upset about this." b. "God always forgives us for our sins." c. "Why do you think you are being punished?" d. "If you feel this way, you should talk to your minister."

a. "You sound very upset about this."

A woman gave birth to an infant boy 10 hours ago. Where does the nurse expect to locate this woman's fundus? a. 1 centimeter above the umbilicus b. 2 centimeters below the umbilicus c. Midway between the umbilicus and the symphysis pubis d. Nonpalpable abdominally

a. 1 centimeter above the umbilicus

Which statement regarding abnormal uterine bleeding (AUB) is most accurate? a. AUB is most commonly caused by anovulation. b. AUB most often occurs in middle age. c. The diagnosis of AUB should be the first consideration for abnormal menstrual bleeding. d. Steroids are the most effective medical treatment for AUB.

a. AUB is most commonly caused by anovulation.

A woman has requested an epidural block for her pain. She is 5 cm dilated and 100% effaced. The baby is in a vertex position and is engaged. The nurse increases the woman's IV fluid for a preprocedural bolus. Before the initiation of the epidural, the woman should be informed regarding the disadvantages of an epidural block. Which concerns should the nurse share with this client? (Select all that apply.) a. Ability to move freely is limited. b. Orthostatic hypotension and dizziness may occur. c. Gastric emptying is not delayed. d. Higher body temperature may occur. e. Blood loss is not excessive.

a. Ability to move freely is limited. b. Orthostatic hypotension and dizziness may occur. d. Higher body temperature may occur.

Which term is an accurate description of the process by which people retain some of their own culture while adopting the practices of the dominant society? a. Acculturation b. Assimilation c. Ethnocentrism d. Cultural relativism

a. Acculturation

Which pictorial tool can assist the nurse in assessing the aspects of family life related to health care? a. Genogram b. Ecomap c. Life-cycle model d. Human development wheel

a. Genogram

Which societal factors have a strong influence on parental response to their infant? (Select all that apply.) a. An adolescent mother's egocentricity and unmet developmental needs interfere with her ability to parent effectively. b. An adolescent mother is likely to use less verbal instruction, be less responsive, and interact less positively than other mothers. c. Adolescent mothers have a higher documented incidence of child abuse. d. Mothers older than 35 years of age often deal with more stress related to work and career issues, as well as decreasing libido. e. Relationships between adolescent mothers and fathers are more stable than older adults.

a. An adolescent mother's egocentricity and unmet developmental needs interfere with her ability to parent effectively. b. An adolescent mother is likely to use less verbal instruction, be less responsive, and interact less positively than other mothers. d. Mothers older than 35 years of age often deal with more stress related to work and career issues, as well as decreasing libido.

A laboring woman is reclining in the supine position. What is the most appropriate nursing action at this time? a. Ask her to turn to one side. b. Elevate her feet and legs. c. Take her blood pressure. d. Determine whether fetal tachycardia is present.

a. Ask her to turn to one side.

As a nurse in the emergency department, you are caring for a patient who is exhibiting signs of depression. What is a priority nursing intervention you should perform for this patient? a. Assess for depression and ask directly about suicide thoughts. b. Ask the care provider to prescribe blood lab work to assess for depression. c. Focus on the presenting problems and refer the patient for a mental health evaluation. d. Interview the patient's family to identify their concerns about the patient's behaviors.

a. Assess for depression and ask directly about suicide thoughts.

What is the highest priority nursing intervention when admitting a pregnant woman who has experienced a bleeding episode in late pregnancy? a. Assessing fetal heart rate (FHR) and maternal vital signs b. Performing a venipuncture for hemoglobin and hematocrit levels c. Placing clean disposable pads to collect any drainage d. Monitoring uterine contractions

a. Assessing fetal heart rate (FHR) and maternal vital signs

Which nursing intervention should be immediately performed after the forceps-assisted birth of an infant? a. Assessing the infant for signs of trauma b. Administering prophylactic antibiotic agents to the infant c. Applying a cold pack to the infant's scalp d. Measuring the circumference of the infant's head

a. Assessing the infant for signs of trauma

New parents express concern that because of the mother's emergency cesarean birth under general anesthesia, they did not have the opportunity to hold and bond with their daughter immediately after her birth. Which information should the nurse's response convey? a. Attachment, or bonding, is a process that occurs over time and does not require early contact. b. Time immediately after birth is a critical period for humans. c. Early contact is essential for optimal parent-infant relationships. d. These new parents should just be happy that the infant is healthy.

a. Attachment, or bonding, is a process that occurs over time and does not require early contact.

After delivery, excess hypertrophied tissue in the uterus undergoes a period of self-destruction. What is the correct term for this process? a. Autolysis b. Subinvolution c. Afterpains d. Diastasis

a. Autolysis

Which suggestions are appropriate for a client who reports experiencing hot flashes? (Select all that apply.) a. Avoiding caffeine. b. Drinking a glass of wine to relax. c. Wearing layered clothing. d. Drinking ice water. e. Consuming spicy foods

a. Avoiding caffeine. c. Wearing layered clothing. d. Drinking ice water.

Which order should the nurse expect for a client admitted with a threatened abortion? a. Bed rest b. Administration of ritodrine IV c. Nothing by mouth (nil per os [NPO]) d. Narcotic analgesia every 3 hours, as needed

a. Bed rest

The nurse is teaching an adolescent about the use of tretinoin (Retin-A). What should the nurse include in the teaching session? (Select all that apply.) a. Begin with a pea-sized dot of medication. b. Apply additional medication to the throat. c. Use sunscreen daily and avoid the sun when possible. d. Divide the medication into the three main areas of the face. e. Apply the medication immediately after washing the face.

a. Begin with a pea-sized dot of medication. c. Use sunscreen daily and avoid the sun when possible. d. Divide the medication into the three main areas of the face.

With regard to medications, herbs, boosters, and other substances normally encountered by pregnant women, what is important for the nurse to be aware of? a. Both prescription and over-the-counter (OTC) drugs that otherwise are harmless can be made hazardous by metabolic deficiencies of the fetus. b. The greatest danger of drug-caused developmental deficits in the fetus is observed in the final trimester. c. Killed-virus vaccines (e.g., tetanus) should not be administered during pregnancy, but live-virus vaccines (e.g., measles) are permissible. d. No convincing evidence exists that secondhand smoke is potentially dangerous to the fetus.

a. Both prescription and over-the-counter (OTC) drugs that otherwise are harmless can be made hazardous by metabolic deficiencies of the fetus.

A perimenopausal client has arrived for her annual gynecologic examination. Which preexisting condition would be extremely important for the nurse to identify during a discussion regarding the risks and benefits of hormone therapy? a. Breast cancer b. Vaginal and urinary tract atrophy c. Osteoporosis d. Arteriosclerosis

a. Breast cancer

The clinic nurse is assessing an adolescent on a topical antibacterial agent. The nurse should assess for which side effects that can be seen with topical antibacterial agents? (Select all that apply.) a. Burning b. Dryness c. Dry eyes d. Erythema e. Nasal irritation

a. Burning b. Dryness d. Erythema

Which medications can be taken by postmenopausal women to treat and/or prevent osteoporosis? (Select all that apply.) a. Calcium b. NSAIDs c. Alendronate d. Risedronate sodium e. Calcitonin

a. Calcium c. Alendronate d. Risedronate sodium e. Calcitonin

A postmenopausal client is experiencing low back and pelvic pain, fatigue, and bloody vaginal discharge. What laboratory tests would the nurse expect to see ordered for this client if endometrial cancer is suspected? (Select all that apply.) a. Cancer antigen-125 (CA-125) b. White blood cell (WBC) count c. Hemoglobin and hematocrit (H&H) d. International normalized ratio (INR) e. Prothrombin time (PT)

a. Cancer antigen-125 (CA-125) c. Hemoglobin and hematocrit (H&H)

While evaluating an external monitor tracing of a woman in active labor, the nurse notes that the fetal heart rate (FHR) for five sequential contractions begins to decelerate late in the contraction, with the nadir of the decelerations occurring after the peak of the contraction. What is the nurse's first priority? a. Change the woman's position. b. Notify the health care provider. c. Assist with amnioinfusion d. Insert a scalp electrode.

a. Change the woman's position.

Which information regarding the procedures and criteria for admitting a woman to the hospital labor unit is important for the nurse to understand? a. Client is considered in active labor when she arrives at the facility with contractions. b. Client can have only her male partner or predesignated doula with her at assessment. c. Children are not allowed on the labor unit. d. Non-English speaking client must bring someone to translate.

a. Client is considered in active labor when she arrives at the facility with contractions.

A nurse is caring for four postoperative clients who each had a total abdominal hysterectomy. Which client should the nurse assess first upon initial rounding? a. Client who has had two saturated perineal pads in the last 2 hours b. Client with a temperature of 99 F and blood pressure of 115/73 mm Hg c. Client who has pain of 4 on a scale of 0 to 10 d. Client with a urinary catheter output of 150 mL in the last 3 hours

a. Client who has had two saturated perineal pads in the last 2 hours

Which client would not be a suitable candidate for internal electronic fetal monitoring (EFM)? a. Client who still has intact membranes b. Woman whose fetus is well engaged in the pelvis c. Pregnant woman who has a comorbidity of obesity d. Client whose cervix is dilated to 4 to 5 cm

a. Client who still has intact membranes

What are the legal responsibilities of the perinatal nurses? a. Correctly interpreting fetal heartrate (FHR) patterns, initiating appropriate nursing interventions, and documenting the outcomes b. Greeting the client on arrival, assessing her status, and starting an IV line c. Applying the external fetal monitor and notifying the health care provider d. Ensuring that the woman is comfortable

a. Correctly interpreting fetal heartrate (FHR) patterns, initiating appropriate nursing interventions, and documenting the outcomes

Which benefits are most associated with use of telehealth technologies? (Select all that apply.) a. Cost savings for patients b. Maximize care management c. Access to services for patients in rural areas d. Prompt reimbursement by third-party payers e. Rapid development of trusting relationships with patients

a. Cost savings for patients b. Maximize care management c. Access to services for patients in rural areas

A client is experiencing back labor and reports intense pain in her lower back. Which measure provided by the woman's labor coach would best support this woman in labor? a. Counterpressure against the sacrum b. Pant-blow (breaths and puffs) breathing techniques c. Effleurage d. Conscious relaxation or guided imagery

a. Counterpressure against the sacrum

While developing an intrapartum care plan for the client in early labor, which psychosocial factors would the nurse recognize upon the client's pain experience? (Select all that apply.) a. Culture b. Anxiety and fear c. Previous experiences with pain d. Intervention of caregivers e. Support systems

a. Culture b. Anxiety and fear c. Previous experiences with pain e. Support systems

When assessing a woman in the first stage of labor, which clinical finding will alert the nurse that uterine contractions are effective? a. Dilation of the cervix b. Descent of the fetus to -2 station c. Rupture of the amniotic membranes (ROM) d. Increase in bloody show

a. Dilation of the cervix

In caring for an immediate postpartum client, the nurse notes petechiae and oozing from her intravenous (IV) site. The client would be closely monitored for which clotting disorder? a. Disseminated intravascular coagulation (DIC) b. Amniotic fluid embolism (AFE) c. Hemorrhage d. HELLP syndrome

a. Disseminated intravascular coagulation (DIC)

Part of the nurse's role is assisting with pushing and positioning. Which guidance should the nurse provide to her client in active labor? a. Encourage the woman's cooperation in avoiding the supine position. b. Advise the woman to avoid the semi-Fowler position. c. Encourage the woman to hold her breath and tighten her abdominal muscles to produce a vaginal response. d. Instruct the woman to open her mouth and close her glottis, letting air escape after the push.

a. Encourage the woman's cooperation in avoiding the supine position.

When managing the care of a woman in the second stage of labor, the nurse uses various measures to enhance the progress of fetal descent. Which instruction best describes these measures? a. Encouraging the woman to try various upright positions, including squatting and standing b. Telling the woman to start pushing as soon as her cervix is fully dilated c. Continuing an epidural anesthetic so pain is reduced and the woman can relax d. Coaching the woman to use sustained, 10- to 15-second, closed-glottis bearing-down efforts with each contraction

a. Encouraging the woman to try various upright positions, including squatting and standing

A client reports experiencing severe abdominal and pelvic pain around the time of menstruation. This pain has become progressively worse over the last 5 years. She also experiences pain during intercourse and has tried unsuccessfully to become pregnant for the past 18 months. To which condition are these symptoms most likely related? a. Endometriosis b. Premenstrual Syndrome (PMS) c. Primary dysmenorrhea d. Secondary dysmenorrhea

a. Endometriosis

What method is the most commonly used in completed suicides? a. Firearms b. Drug overdose c. Self-inflicted laceration d. Carbon monoxide poisoning

a. Firearms

Which stage of labor varies the most in length? a. First b. Second c. Third d. Fourth

a. First

Which description of the four stages of labor is correct for both the definition and the duration? a. First stage: onset of regular uterine contractions to full dilation; less than 1 hour to 20 hours b. Second stage: full effacement to 4 to 5 cm; visible presenting part; 1 to 2 hours c. Third stage: active pushing to birth; 20 minutes (multiparous woman), 50 minutes (nulliparous woman) d. Fourth stage: delivery of the placenta to recovery; 30 minutes to 1 hour

a. First stage: onset of regular uterine contractions to full dilation; less than 1 hour to 20 hours

What is the primary role of the doula during labor? a. Helps the woman perform breathing techniques and provides support to the woman and her partner b. Checks the fetal monitor tracing for effects of the labor process on the fetal heart rate c. Takes the place of the father as a coach and support provider d. Administers pain medications as needed by the woman

a. Helps the woman perform breathing techniques and provides support to the woman and her partner

The nurse is educating a client on the prevention of toxic shock syndrome (TSS). Which statement by the client indicates a lack of understanding? a. I need to change my tampon every 8 hours during the day. b. At night, I should use a feminine pad rather than a tampon. c. If I dont use tampons, I should not get TSS. d. It is best if I wash my hands before inserting the tampon.

a. I need to change my tampon every 8 hours during the day.

The nurse is giving discharge instructions to a client who had a total abdominal hysterectomy. Which statements by the client indicate a need for further teaching? (Select all that apply.) a. I should not have any problems driving to see my mother, who lives 3 hours away. b. Now that I have time off from work, I can return to my exercise routine next week. c. My granddaughter weighs 23 pounds, so I need to refrain from picking her up. d. I will have to limit the times that I climb our stairs at home to morning and night. e. For 1 month, I will need to refrain from sexual intercourse.

a. I should not have any problems driving to see my mother, who lives 3 hours away. b. Now that I have time off from work, I can return to my exercise routine next week.

A nurse caring for a patient diagnosed with major depressive disorder reads in the patient's medical record, "This patient shows vegetative signs of depression." Which nursing diagnoses most clearly relate to this documentation? (Select all that apply.) a. Imbalanced nutrition: less than body requirements b. Chronic low self-esteem c. Sexual dysfunction d. Self-care deficit e. Powerlessness f. Insomnia

a. Imbalanced nutrition: less than body requirements c. Sexual dysfunction d. Self-care deficit f. Insomnia

The nurse expects which maternal cardiovascular finding during labor? a. Increased cardiac output b. Decreased pulse rate c. Decreased white blood cell (WBC) count d. Decreased blood pressure

a. Increased cardiac output

Which concerns regarding parenthood are often expressed by visually impaired mothers? (Select all that apply.) a. Infant safety b. Transportation c. Ability to care for the infant d. Visually missing out e. Needing extra time for parenting activities to accommodate the visual limitations

a. Infant safety b. Transportation d. Visually missing out e. Needing extra time for parenting activities to accommodate the visual limitations

While completing an assessment of a homeless woman, the nurse should be aware of which of the following ailments this client is at a higher risk to develop? (Select all that apply.) a. Infectious diseases b. Chronic illness c. Anemia d. Hyperthermia e. Substance abuse

a. Infectious diseases b. Chronic illness c. Anemia e. Substance abuse

The nurse is caring for a client in early labor. Membranes ruptured approximately 2 hours earlier. This client is at increased risk for which complication? a. Intrauterine infection b. Hemorrhage c. Precipitous labor d. Supine hypotension

a. Intrauterine infection

Which condition is considered a medical emergency that requires immediate treatment? a. Inversion of the uterus b. Hypotonic uterus c. Idiopathic thrombocytopenic purpura (ITP) d. Uterine atony

a. Inversion of the uterus

A client who has undergone a dilation and curettage (D&C)for early pregnancy loss is likely to be discharged the same day. The nurse must ensure that her vital signs are stable, that bleeding has been controlled, and that the woman has adequately recovered from the administration of anesthesia. To promote an optimal recovery, what information should discharge teaching include? (Select all that apply.) a. Iron supplementation b. Resumption of intercourse at 6 weeks post-procedure c. Referral to a support group, if necessary d. Expectation of heavy bleeding for at least 2 weeks e. Emphasizing the need for rest

a. Iron supplementation c. Referral to a support group, if necessary e. Emphasizing the need for rest

Which description most accurately describes the augmentation of labor? a. Is part of the active management of labor that is instituted when the labor process is unsatisfactory b. Relies on more invasive methods when oxytocin and amniotomy have failed c. Is a modern management term to cover up the negative connotations of forceps-assisted birth d. Uses vacuum cups

a. Is part of the active management of labor that is instituted when the labor process is unsatisfactory

A patient was diagnosed with seasonal affective disorder (SAD). During which month would this patient's symptoms be most acute? a. January b. April c. June d. September

a. January

What is the most critical nursing action in caring for the newborn immediately after the birth? a. Keeping the airway clear b. Fostering parent-newborn attachment c. Drying the newborn and wrapping the infant in a blanket d. Administering eye drops and vitamin K

a. Keeping the airway clear

Which changes take place in the woman's reproductive system, days or even weeks before the commencement of labor? (Select all that apply.) a. Lightening b. Exhaustion c. Bloody show d. Rupture of membranes e. Decreased Fetal movement

a. Lightening c. Bloody show d. Rupture of membranes

The nurse and the patient are conversing face to face. What communication technique is being demonstrated? a. Linguistic b. Paralinguistic c. Explicit d. Metacommunication

a. Linguistic

In assessing the immediate condition of the newborn after birth, a sample of cord blood may be a useful adjunct to the Apgar score. Cord blood is then tested for pH, carbon dioxide, oxygen, and base deficit or excess. Which clinical situation warrants this additional testing? (Select all that apply.) a. Low 5-minute Apgar score b. Intrauterine growth restriction (IUGR) c. Maternal thyroid disease d. Intrapartum fever e. Vacuum extraction

a. Low 5-minute Apgar score b. Intrauterine growth restriction (IUGR) c. Maternal thyroid disease d. Intrapartum fever

Which medications are used to manage postpartum hemorrhage (PPH)? (Select all that apply.) a. Oxytocin b. Methergine c. Terbutaline d. Hemabate e. Magnesium sulfate

a. Oxytocin b. Methergine d. Hemabate

A patient diagnosed with major depressive disorder does not interact with others except when addressed, and then only in monosyllables. The nurse wants to show nonjudgmental acceptance and support for the patient. Which communication technique will be effective? a. Make observations. b. Ask the patient direct questions. c. Phrase questions to require yes or no answers. d. Frequently reassure the patient to reduce guilt feelings.

a. Make observations.

A married couple lives in a single-family house with their newborn son and the husband's daughter from a previous marriage. Based on this information, what family form best describes this family? a. Married-blended family b. Extended family c. Nuclear family d. Same-sex family

a. Married-blended family

Nursing care measures are commonly offered to women in labor. Which nursing measure reflects the application of the gate-control theory? a. Massaging the woman's back. b. Changing the woman's position. c. Giving the prescribed medication. d. Encouraging the woman to rest between contractions.

a. Massaging the woman's back.

Which clinical finding or intervention might be considered the rationale for fetal tachycardia to occur? a. Maternal fever b. Umbilical cord prolapse c. Regional anesthesia d. Magnesium sulfate administration

a. Maternal fever

With one exception, the safest pregnancy is one during which the woman is drug and alcohol free. What is the optimal treatment for women addicted to opioids? a. Methadone b. Detoxification c. Smoking cessation d. 4 Ps Plus

a. Methadone

As the United States and Canada continue to become more culturally diverse, recognizing a wide range of varying cultural beliefs and practices is increasingly important for the nursing staff. A client is from which country if she requests to have the baby's father in attendance? a. Mexico b. China c. Iran d. India

a. Mexico

The nurse should be aware of which important information regarding nerve block analgesia and anesthesia? a. Most local agents are chemically related to cocaine and end in the suffix -caine. b. Local perineal infiltration anesthesia is effective when epinephrine is added, but it can be injected only once. c. Pudendal nerve block is designed to relieve the pain from uterine contractions. d. Pudendal nerve block, if performed correctly, does not significantly lessen the bearing-down reflex.

a. Most local agents are chemically related to cocaine and end in the suffix -caine.

Rho immune globulin will be ordered postpartum if which situation occurs? a. Mother Rh-, baby Rh+ b. Mother Rh-, baby Rh- c. Mother Rh+, baby Rh+ d. Mother Rh+, baby Rh-

a. Mother Rh-, baby Rh+

A client's household consists of her husband, his mother, and another child. To which family configuration does this client belong? a. Multigenerational family b. Single-parent family c. Married-blended family d. Nuclear family

a. Multigenerational family

When the infant's behaviors and characteristics call forth a corresponding set of maternal behaviors and characteristics, what is the correct term for this behavior? a. Mutuality b. Bonding c. Claiming d. Acquaintance

a. Mutuality

Several delivery changes in the integumentary system that appear during pregnancy disappear after birth, although not always completely. What change is almost certain to be completely reversed? a. Nail brittleness b. Darker pigmentation of the areolae and linea nigra c. Striae gravidarum on the breasts, abdomen, and thighs d. Spider nevi

a. Nail brittleness

Emergency conditions during labor that would require immediate nursing intervention can arise with startling speed. Which situations are examples of such an emergency? (Select all that apply.) a. Nonreassuring or abnormal fetal heart rate (FHR) pattern b. Inadequate uterine relaxation c. Vaginal bleeding d. Prolonged second stage e. Prolapse of the cord

a. Nonreassuring or abnormal fetal heart rate (FHR) pattern b. Inadequate uterine relaxation c. Vaginal bleeding e. Prolapse of the cord

While talking with a patient diagnosed with major depressive disorder, a nurse notices the patient is unable to maintain eye contact. The patient's chin lowers to the chest. The patient looks at the floor. Which aspect of communication has the nurse assessed? a. Nonverbal communication b. A message filter c. A cultural barrier d. Social skills

a. Nonverbal communication

The woman's family members are present when the nurse arrives for a postpartum and newborn visit. What should the nurse do? a. Observe the family members' interactions with the newborn and one another. b. Ask the woman to meet with her and the baby alone. c. Perform a brief assessment on all family members who are present. d. Reschedule the visit for another time so that the mother and infant can be privately assessed.

a. Observe the family members' interactions with the newborn and one another.

Under which circumstance should the nurse assist the laboring woman into a hands-and-knees position? a. Occiput of the fetus is in a posterior position. b. Fetus is at or above the ischial spines. c. Fetus is in a vertex presentation. d. Membranes have ruptured.

a. Occiput of the fetus is in a posterior position.

A patient diagnosed with major depressive disorder shows vegetative signs of depression. Which nursing actions should be implemented? (Select all that apply.) a. Offer laxatives if needed. b. Monitor food and fluid intake. c. Provide a quiet sleep environment. d. Eliminate all daily caffeine intake. e. Restrict intake of processed foods.

a. Offer laxatives if needed. b. Monitor food and fluid intake. c. Provide a quiet sleep environment.

Lacerations of the cervix, vagina, or perineum are also causes of postpartum hemorrhage (PPH). Which factors influence the causes and incidence of obstetric lacerations of the lower genital tract? (Select all that apply.) a. Operative and precipitate births b. Adherent retained placenta c. Abnormal presentation of the fetus d. Congenital abnormalities of the maternal soft tissue e. Previous scarring from infection

a. Operative and precipitate births c. Abnormal presentation of the fetus d. Congenital abnormalities of the maternal soft tissue e. Previous scarring from infection

What is the correct placement of the tocotransducer for effective electronic fetal monitoring (EFM)? a. Over the uterine fundus b. On the fetal scalp c. Inside the uterus d. Over the mother's lower abdomen

a. Over the uterine fundus

Screening questions for alcohol and drug abuse should be included in the overall assessment during the first prenatal visit for all women. The 4 Ps Plus is a screening tool specifically designed to identify the need for a more in-depth assessment. Which are the correct components of the 4 Ps Plus? (Select all that apply.) a. Parents b. Partner c. Present d. Past e. Pregnancy

a. Parents b. Partner d. Past e. Pregnancy

Because of its size and rigidity, the fetal head has a major effect on the birth process. Which bones comprise the structure of the fetal skull? (Select all that apply.) a. Parietal b. Temporal c. Fontanel d. Occipital e. Femoral

a. Parietal b. Temporal d. Occipital

At least five factors affect the process of labor and birth. These are easily remembered as the five Ps. Which factors are included in this process? (Select all that apply.) a. Passenger b. Passageway c. Powers d. Pressure e. Psychologic response

a. Passenger b. Passageway c. Powers e. Psychologic response

The nurse suspects that a client has early signs of ectopic pregnancy. The nurse should be observing the client for which signs or symptoms? (Select all that apply.) a. Pelvic pain b. Abdominal pain c. Unanticipated heavy bleeding d. Vaginal spotting or light bleeding e. Missed period

a. Pelvic pain b. Abdominal pain d. Vaginal spotting or light bleeding e. Missed period

A nurse caring for a woman in labor should understand that absent or minimal variability is classified as either abnormal or indeterminate. Which condition related to decreased variability is considered benign? a. Periodic fetal sleep state b. Extreme prematurity c. Fetal hypoxemia d. Preexisting neurologic injury

a. Periodic fetal sleep state

A pregnant woman's amniotic membranes have ruptured. A prolapsed umbilical cord is suspected. What intervention would be the nurse's highest priority? a. Placing the woman in the knee-chest position b. Covering the cord in sterile gauze soaked in saline c. Preparing the woman for a cesarean birth d. Starting oxygen by face mask

a. Placing the woman in the knee-chest position

Which statement by the nurse can assist a new father in his transition to parenthood? a. Pointing out that the infant turned at the sound of his voice b. Encouraging him to go home to get some sleep c. Telling him to tape the infant's diaper a different way d. Suggesting that he let the infant sleep in the bassinet

a. Pointing out that the infant turned at the sound of his voice

The nurse should be aware of which physiologic effect of labor pain? a. Predominant pain of the first stage of labor is visceral pain that is in the lower portion of the abdomen. b. Referred pain is the extreme discomfort experienced between contractions. c. Somatic pain of the second stage of labor is more generalized and related to fatigue. d. Pain during the third stage is a somewhat milder version of the pain experienced during the second stage.

a. Predominant pain of the first stage of labor is visceral pain that is in the lower portion of the abdomen.

What is the correct definition of a spontaneous termination of a pregnancy (abortion)? a. Pregnancy is less than 20 weeks. b. Fetus weighs less than 1000 g. c. Products of conception are passed intact. d. No evidence exists of intrauterine infection.

a. Pregnancy is less than 20 weeks.

According to research, which risk factor for postpartum depression (PPD) is likely to have the greatest effect on the client postpartum? a. Prenatal depression b. Single-mother status c. Low socioeconomic status d. Unplanned or unwanted pregnancy

a. Prenatal depression

Women who have participated in childbirth education classes often bring a birth plan with them to the hospital. Which items might this plan include? (Select all that apply.) a. Presence of companions b. Clothing to be worn c. Care and handling of the newborn d. Medical interventions e. Date of delivery

a. Presence of companions b. Clothing to be worn c. Care and handling of the newborn d. Medical interventions

What is a priority goal in the postpartum care of an adolescent mother? a. Prevention of subsequent pregnancies b. Ensuring that the father of the baby cares for the child c. Returning the mother to a prepregnancy lifestyle d. Facilitating formula feeding to minimize interruptions

a. Prevention of subsequent pregnancies

A 21-year-old client reports experiencing severe pain immediately after the commencement of her menses. Which gynecologic condition is the most likely cause of this client's presenting symptoms? a. Primary dysmenorrhea b. Secondary dysmenorrhea c. Dyspareunia d. Endometriosis

a. Primary dysmenorrhea

What is the nurse's understanding of the appropriate role of primary and secondary powers? a. Primary powers are responsible for the effacement and dilation of the cervix. b. Effacement is generally well ahead of dilation in women giving birth for the first time; they are closer together in subsequent pregnancies. c. Scarring of the cervix caused by a previous infection or surgery may make the delivery a bit more painful, but it should not slow or inhibit dilation. d. Pushing in the second stage of labor is more effective if the woman can breathe deeply and control some of her involuntary needs to push, as the nurse directs.

a. Primary powers are responsible for the effacement and dilation of the cervix.

What are the complications and risks associated with cesarean births? (Select all that apply.) a. Pulmonary edema b. Wound dehiscence c. Hemorrhage d. Urinary tract infections e. Fetal injuries

a. Pulmonary edema b. Wound dehiscence c. Hemorrhage d. Urinary tract infections e. Fetal injuries

If a woman is at risk for thrombus but is not ready to ambulate, which nursing intervention would the nurse include in the plan of care? (Select all that apply.) a. Putting her in antiembolic stockings b. Having her flex, extend, and rotate her feet, ankles, and legs c. Having her sit in a chair for at 30 uninterrupted minutes d. Immediately notifying the healthcare provider when a positive Homans sign occurs e. Applying sequential compression devices as prescribed

a. Putting her in antiembolic stockings b. Having her flex, extend, and rotate her feet, ankles, and legs d. Immediately notifying the healthcare provider when a positive Homans sign occurs e. Applying sequential compression devices as prescribed

A pregnant woman at 18 weeks of gestation calls the clinic to report that she has been experiencing occasional backaches of mild-to-moderate intensity. Which intervention should the nurse recommend? a. Kegel exercises b. Pelvic rock exercises c. Softer mattress d. Bed rest for 24 hours

b. Pelvic rock exercises

In recovery, if a woman is asked to either raise her legs (knees extended) off the bed or flex her knees, and then place her feet flat on the bed and raise her buttocks well off the bed, the purpose of this exercise is to assess what? a. Recovery from epidural or spinal anesthesia b. Hidden bleeding underneath her c. Flexibility d. Whether the woman is a candidate to go home after 6 hours

a. Recovery from epidural or spinal anesthesia

A 28-year-old client is diagnosed with endometriosis and is experiencing severe symptoms. Which actions by the nurse are the most appropriate at this time? (Select all that apply.) a. Reduce the pain by low-level heat. b. Discuss the high risk of infertility with this diagnosis. c. Relieve anxiety by relaxation techniques and education. d. Discuss in detail the side effects of laparoscopic surgery. e. Suggest resources such as the Endometriosis Association

a. Reduce the pain by low-level heat. c. Relieve anxiety by relaxation techniques and education. e. Suggest resources such as the Endometriosis Association

A 13-year-old boy comes to the school nurse complaining of sudden and severe scrotal pain. He denies any trauma to the scrotum. What is the most appropriate nursing action? a. Refer him for immediate medical evaluation. b. Administer analgesics and recommend scrotal support. c. Apply an ice bag and observe for increasing pain. d. Reassure the adolescent that occasional pain is common with the changes of puberty.

a. Refer him for immediate medical evaluation.

Nurses are in an ideal position to educate clients who experience Premenstrual dysphoric disorder (PMDD). What self-help activities have been documented as helpful in alleviating the symptoms of PMDD? (Select all that apply.) a. Regular exercise b. Improved nutrition c. Daily glass of wine d. Smoking cessation e. Oil of evening primrose

a. Regular exercise b. Improved nutrition d. Smoking cessation e. Oil of evening primrose

Which physiologic factors are reliable indicators of impending shock from postpartum hemorrhage? (Select all that apply.) a. Respirations b. Skin condition c. Blood pressure d. Level of consciousness e. Urinary output

a. Respirations b. Skin condition d. Level of consciousness e. Urinary output

The client has delivered by urgent caesarean birth for fetal compromise. Umbilical cord gases were obtained for acid-base determination. The pH is 6.9, partial pressure of carbon dioxide (PCO2) is elevated, and the base deficit is 11 mmol/L. What type of acidemia is displayed by the infant? a. Respiratory b. Metabolic c. Mixed d. Turbulent

a. Respiratory

Despite warnings, prenatal exposure to alcohol continues to far exceed exposure to illicit drugs. Which condition is rarely associated with fetal alcohol syndrome (FAS)? a. Respiratory conditions b. Intellectual impairment c. Neural development disorder d. Alcohol-related birth defects (ARBDs)

a. Respiratory conditions

Which technique will best communicate to a patient that the nurse is interested in listening? a. Restating a feeling or thought the patient has expressed. b. Asking a direct question, such as "Did you feel angry?" c. Making a judgment about the patient's problem. d. Saying, "I understand what you're saying."

a. Restating a feeling or thought the patient has expressed.

Which technique is an adequate means of controlling the birth of the fetal head during delivery in a vertex presentation? a. Ritgen maneuver b. Fundal pressure c. Lithotomy position d. De Lee apparatus

a. Ritgen maneuver

The laboratory results for a postpartum woman are as follows: blood type, A; Rh status, positive; rubella titer, 1:8 (enzyme immunoassay [EIA] 0.8); hematocrit, 30%. How should the nurse best interpret these data? a. Rubella vaccine should be administered. b. Blood transfusion is necessary. c. Rh immune globulin is necessary within 72 hours of childbirth. d. Kleihauer-Betke test should be performed.

a. Rubella vaccine should be administered.

The induction of labor is considered an acceptable obstetric procedure if it is in the best interest to deliver the fetus. The charge nurse on the labor and delivery unit is often asked to schedule clients for this procedure and therefore must be cognizant of the specific conditions appropriate for labor induction. What are appropriate indications for induction? (Select all that apply?) a. Rupture of membranes at or near term b. Convenience of the woman or her physician c. Chorioamnionitis (inflammation of the amniotic sac) d. Postterm pregnancy e. Fetal death

a. Rupture of membranes at or near term c. Chorioamnionitis (inflammation of the amniotic sac) d. Postterm pregnancy e. Fetal death

Which alterations in the perception of pain by a laboring client should the nurse understand? a. Sensory pain for nulliparous women is often greater than for multiparous women during early labor. b. Affective pain for nulliparous women is usually less than for multiparous women throughout the first stage of labor. c. Women with a history of substance abuse experience more pain during labor. d. Multiparous women have more fatigue from labor and therefore experience more pain.

a. Sensory pain for nulliparous women is often greater than for multiparous women during early labor.

Which behavior indicates that a woman is "seeking safe passage" for herself and her infant? a. She keeps all prenatal appointments. b. She "eats for two." c. She slowly drives her car. d. She wears only low heeled shoes

a. She keeps all prenatal appointments.

Which documentation for a patient diagnosed with major depressive disorder indicates the treatment plan was effective? a. Slept 6 hours uninterrupted. Sang with activity group. Anticipates seeing grandchild. b. Slept 10 hours uninterrupted. Attended craft group; stated "project was a failure, just like me." c. Slept 5 hours with brief interruptions. Personal hygiene adequate with assistance. Weight loss of 1 pound. d. Slept 7 hours uninterrupted. Preoccupied with perceived inadequacies. States, "I feel tired all the time."

a. Slept 6 hours uninterrupted. Sang with activity group. Anticipates seeing grandchild.

During a well-child visit, the nurse practitioner provides guidance about promoting healthy eating in a child who is overweight. What does the nurse advise? a. Slow down eating meals. b. Avoid between-meal snacks. c. Include low-fat foods in meals. d. Use foods that child likes as special treats.

a. Slow down eating meals.

Which breathing pattern should the nurse support for the woman and her coach during the latent phase of the first stage of labor if the couple has attended childbirth preparation classes? a. Slow-paced breathing b. Deep abdominal breathing c. Modified-paced breathing d. Patterned-paced breathing

a. Slow-paced breathing

The nurse is taking the history of a 24-year-old client diagnosed with cervical cancer. What possible risk factors would the nurse assess? (Select all that apply.) a. Smoking b. Multiple sexual partners c. Poor diet d. Nulliparity e. Younger than 18 at first intercourse

a. Smoking b. Multiple sexual partners c. Poor diet e. Younger than 18 at first intercourse

A client has requested information regarding alternatives to hormonal therapy for menopausal symptoms. Which current information should the nurse provide to the client? (Select all that apply.) a. Soy b. Vitamin C c. Vitamin K d. Vitamin E e. Vitamin A

a. Soy d. Vitamin E

Which factors influence cervical dilation? (Select all that apply.) a. Strong uterine contractions b. Force of the presenting fetal part against the cervix c. Size of the woman d. Pressure applied by the amniotic sac e. Scarring of the cervix

a. Strong uterine contractions b. Force of the presenting fetal part against the cervix d. Pressure applied by the amniotic sac e. Scarring of the cervix

The nurse is admitting a prenatal patient for diagnostic testing. While eliciting the psychosocial history, the nurse learns the patient smokes a pack of cigarettes daily, drinks a cup of cappuccino with breakfast, has smoked marijuana in the remote past, and is a social drinker. Which action should the nurse first take? a. Strongly advise immediate tobacco cessation b. Elimination of all caffeinated beverages c. Serum and urine testing for drug use and alcohol use d. Referral to a 12-step program

a. Strongly advise immediate tobacco cessation

What is the most common reason for late postpartum hemorrhage (PPH)? a. Subinvolution of the uterus b. Defective vascularity of the decidua c. Cervical lacerations d. Coagulation disorders

a. Subinvolution of the uterus

Which system responses would the nurse recognize as being related to prostaglandin (PGF2) release? (Select all that apply) a. Systemic responses b. Gastrointestinal system c. Central nervous system d. CHOICE BLANK e. Genitourinary system

a. Systemic responses b. Gastrointestinal system c. Central nervous system

A woman gave birth to a 7-pound, 6-ounce infant girl 1 hour ago. The birth was vaginal and the estimated blood loss (EBL) was 1500 ml. When evaluating the woman's vital signs, which finding would be of greatest concern to the nurse? a. Temperature 37.9° C, heart rate 120 beats per minute (bpm), respirations 20 breaths per minute, and blood pressure 90/50 mm Hg b. Temperature 37.4° C, heart rate 88 bpm, respirations 36 breaths per minute, and blood pressure 126/68 mm Hg c. Temperature 38° C, heart rate 80 bpm, respirations 16 breaths per minute, and blood pressure 110/80 mm Hg d. Temperature 36.8° C, heart rate 60 bpm, respirations 18 breaths per minute, and blood pressure 140/90 mm Hg

a. Temperature 37.9° C, heart rate 120 beats per minute (bpm), respirations 20 breaths per minute, and blood pressure 90/50 mm Hg

A woman has just moved to the United States from Mexico. She is 3 months pregnant and has arrived for her first prenatal visit. During her assessment interview, the nurse learns that the client has not had any immunizations. Which immunizations should she receive at this point in her pregnancy? (Select all that apply.) a. Tetanus b. Diphtheria c. Chickenpox d. Rubella e. Hepatitis B

a. Tetanus b. Diphtheria e. Hepatitis B

What is important for the nurse to recognize regarding the new father and his acceptance of the pregnancy and preparation for childbirth? a. The father goes through three phases of acceptance of his own. b. The father's attachment to the fetus cannot be as strong as that of the mother because it does not start until after the birth. c. In the last 2 months of pregnancy, most expectant fathers suddenly get very protective of their established lifestyle and resist making changes to the home. d. Typically, men remain ambivalent about fatherhood right up to the birth of their child.

a. The father goes through three phases of acceptance of his own.

When assessing a multiparous woman who has just given birth to an 8-pound boy, the nurse notes that the woman's fundus is firm and has become globular in shape. A gush of dark red blood comes from her vagina. What is the nurse's assessment of the situation? a. The placenta has separated. b. A cervical tear occurred during the birth. c. The woman is beginning to hemorrhage. d. Clots have formed in the upper uterine segment.

a. The placenta has separated.

A primiparous woman is to be discharged from the hospital the following day with her infant girl. Which behavior indicates a need for further intervention by the nurse before the woman can be discharged? a. The woman is weepy and asks to postpone learning about infant care. b. The woman continues to hold and cuddle her infant after she has fed her. c. The woman asks for several magazines to read while her infant sleeps. d. The woman changes her infant's diaper and then shows the nurse the contents of the diaper.

a. The woman is weepy and asks to postpone learning about infant care.

A pregnant 15-year-old adolescent tells the nurse that she did not use any form of contraception because she was afraid her parents would find out. The nurse should recognize what? a. This is a frequent reason given by adolescents. b. This suggests a poor parentchild relationship. c. This is not a good reason to not get contraception. d. This indicates that the adolescent is unaware of her legal rights.

a. This is a frequent reason given by adolescents.

Women who are obese are at risk for several complications during pregnancy and birth. Which of these would the nurse anticipate with an obese client? (Select all that apply.) a. Thromboembolism b. Cesarean birth c. Wound infection d. Breech presentation e. Hypertension

a. Thromboembolism b. Cesarean birth c. Wound infection e. Hypertension

A hospital has several different perineal pads available for use. A nurse is observed soaking several of them and writing down what is observed. What goal is the nurse attempting to achieve by performing this practice? a. To improve the accuracy of blood loss estimation, which usually is a subjective assessment b. To determine which pad is best c. To demonstrate that other nurses usually underestimate blood loss d. To reveal which brand of pad is more absorbent

a. To improve the accuracy of blood loss estimation, which usually is a subjective assessment

What is the focus of priority nursing interventions for the period immediately after electroconvulsive therapy (ECT) treatment? a. Nutrition and hydration b. Supporting physiological stability c. Reducing disorientation and confusion d. Assisting the patient to identify and test negative thoughts

b. Supporting physiological stability

A woman in preterm labor at 30 weeks of gestation receives two 12-mg intramuscular (IM) doses of betamethasone. What is the purpose of this pharmacologic intervention? a. To stimulate fetal surfactant production b. To reduce maternal and fetal tachycardia associated with ritodrine administration c. To suppress uterine contractions d. To maintain adequate maternal respiratory effort and ventilation during magnesium sulfate therapy

a. To stimulate fetal surfactant production

The nurse should be aware of which information related to a woman's intake and output during labor? a. Traditionally, restricting the laboring woman to clear liquids and ice chips is being challenged because regional anesthesia is used more often than general anesthesia. b. Intravenous (IV) fluids are usually necessary to ensure that the laboring woman stays hydrated. c. Routine use of an enema empties the rectum and is very helpful for producing a clean, clear delivery. d. When a nulliparous woman experiences the urge to defecate, it often means birth will quickly follow.

a. Traditionally, restricting the laboring woman to clear liquids and ice chips is being challenged because regional anesthesia is used more often than general anesthesia.

A client at 34 weeks of gestation seeks guidance from the nurse regarding personal hygiene. Which information should the nurse provide? a. Tub bathing is permitted even in late pregnancy unless membranes have ruptured. b. The perineum should be wiped from back to front. c. Bubble bath and bath oils are permissible because they add an extra soothing and cleansing action to the bath. d. Expectant mothers should use specially treated soap to cleanse the nipples.

a. Tub bathing is permitted even in late pregnancy unless membranes have ruptured.

What conditions are physical complications of obesity? (Select all that apply.) a. Type 2 diabetes mellitus b. QT interval prolongation c. Fatty liver disease d. Gastrointestinal dysfunction e. Abnormal growth acceleration f. Dental erosion

a. Type 2 diabetes mellitus c. Fatty liver disease e. Abnormal growth acceleration

The transition to parenting for same-sex couples can present unique challenges. How can the nurse foster adjustment to parenting for these clients? (Select all that apply.) a. Use a supplemental feeding device to simulate breastfeeding. b. Allow the partner to cut the cord. c. Gay fathers should meet their new infant soon after the birth mother has recovered. d. Understand that strong social sanctions remain. e. Provide information regarding support groups.

a. Use a supplemental feeding device to simulate breastfeeding. b. Allow the partner to cut the cord. d. Understand that strong social sanctions remain. e. Provide information regarding support groups.

A parent who has a hearing impairment is presented with several challenges in parenting. Which nursing approaches are appropriate for working with hearing-impaired new parents? (Select all that apply.) a. Using devices that transform sound into light b. Assuming that the client knows sign language c. Speaking quickly and loudly d. Ascertaining whether the client can read lips before teaching e. Writing messages that aid in communication

a. Using devices that transform sound into light d. Ascertaining whether the client can read lips before teaching e. Writing messages that aid in communication

A perinatal nurse is caring for a woman in the immediate postbirth period. Assessment reveals that the client is experiencing profuse bleeding. What is the most likely cause for this bleeding? a. Uterine atony b. Uterine inversion c. Vaginal hematoma d. Vaginal laceration

a. Uterine atony

Which signs and symptoms should a woman immediately report to her health care provider? (Select all that apply.) a. Vaginal bleeding b. Rupture of membranes c. Heartburn accompanied by severe headache d. Decreased libido e. Urinary frequency

a. Vaginal bleeding b. Rupture of membranes c. Heartburn accompanied by severe headache

A patient being treated with paroxetine 50 mg po daily reports to the clinic nurse, "I took a few extra tablets earlier today and now I feel bad." Which assessments are most critical? (Select all that apply.) a. Vital signs b. Urinary frequency c. Psychomotor retardation d. Presence of abdominal pain and diarrhea e. Hyperactivity or feelings of restlessness

a. Vital signs d. Presence of abdominal pain and diarrhea e. Hyperactivity or feelings of restlessness

An important distinction in understanding substance abuse is that drug misuse, abuse, and addiction are considered what? a. Voluntary behaviors based on psychosocial needs b. Problems that occur in conjunction with addiction c. Involuntary physiologic responses to the pharmacologic characteristics of drugs d. Legal use of substances for purposes other than medicinal.

a. Voluntary behaviors based on psychosocial needs

The class of drugs known as opioid analgesics (butorphanol, nalbuphine) is not suitable for administration to women with known opioid dependence. The antagonistic activity could precipitate withdrawal symptoms (abstinence syndrome) in both mothers and newborns. Which signs would indicate opioid withdrawal in the mother? (Select all that apply.) a. Yawning, runny nose b. Increase in appetite c. Chills or hot flashes d. Constipation e. Irritability, restlessness

a. Yawning, runny nose c. Chills or hot flashes e. Irritability, restlessness

A client has scheduled brachytherapy sessions and states that she feels as though she is not safe around her family. What is the best response by the nurse? a. You are only reactive when the radioactive implant is in place. b. To be totally safe, it is a good idea to sleep in a separate room. c. It is best to stay a safe distance from friends or family between treatments. d. You should use a separate bathroom from the rest of the family.

a. You are only reactive when the radioactive implant is in place.

A patient is having difficulty making a decision. The nurse has mixed feelings about whether to provide advice. Which principle usually applies? Giving advice a. is rarely helpful. b. fosters independence. c. lifts the burden of personal decision making. d. helps the patient develop feelings of personal adequacy.

a. is rarely helpful.

A patient is experiencing psychomotor agitation associated with major depressive disorder. Which observation would the nurse associate with this symptom? The patient a. paces aimlessly around the room. b. asks the nurse to repeat instructions. c. complains of prickly skin sensations. d. demonstrates slowed verbal responses.

a. paces aimlessly around the room.

A patient became depressed after the last of the family's six children moved out of the home 4 months ago. Select the best initial outcome for the nursing diagnosis Situational low self-esteem related to feelings of abandonment. The patient will a. verbalize realistic positive characteristics about self by (date). b. agree to take an antidepressant medication regularly by (date). c. initiate social interaction with another person daily by (date). d. identify two personal behaviors that alienate others by (date).

a. verbalize realistic positive characteristics about self by (date).

Many first-time parents do not plan on having their parents' help immediately after the newborn arrives. Which statement by the nurse is the most appropriate when counseling new parents regarding the involvement of grandparents? a. "You should tell your parents to leave you alone." b. "Grandparents can help you with parenting skills." c. "Grandparent involvement can be very disruptive to the family." d. "They are getting old. You should let them be involved while they can."

b. "Grandparents can help you with parenting skills."

The nurse is working with a patient diagnosed with posttraumatic stress disorder related to childhood sexual abuse. The patient is crying and states, "I should be over this by now; this happened years ago." Which response(s) by the nurse will facilitate communication? (Select all that apply.) a. "Why do you think you are so upset?" b. "I can see that this situation really bothers you." c. "The abuse you endured is very painful for you." d. "Crying is a way of expressing the hurt you're experiencing." e. "Let's talk about something else, since this subject is upsetting you."

b. "I can see that this situation really bothers you." c. "The abuse you endured is very painful for you." d. "Crying is a way of expressing the hurt you're experiencing."

A patient cries as the nurse explores the patient's feelings about the death of a close friend. The patient sobs, "I shouldn't be crying like this. It happened a long time ago." Which responses by the nurse facilitate communication? (Select all that apply.) a. "Why do you think you are so upset?" b. "I can see that you feel sad about this situation." c. "The loss of a close friend is very painful for you." d. "Crying is a way of expressing the hurt you are experiencing." e. "Let's talk about something else because this subject is upsetting you."

b. "I can see that you feel sad about this situation." c. "The loss of a close friend is very painful for you." d. "Crying is a way of expressing the hurt you are experiencing."

Transcranial Magnetic Stimulation (TCM) is scheduled for a patient diagnosed with major depressive disorder. Which comment by the patient indicates teaching about the procedure was effective? a. "They will put me to sleep during the procedure so I won't know what is happening." b. "I might be a little dizzy or have a mild headache after each procedure." c. "I will be unable to care for my children for about 2 months." d. "I will avoid eating foods that contain tyramine."

b. "I might be a little dizzy or have a mild headache after each procedure."

A patient diagnosed with schizophrenia tells the nurse, "The Central Intelligence Agency is monitoring us through the fluorescent lights in this room. The CIA is everywhere, so be careful what you say." Which response by the nurse is most therapeutic? a. "Let's talk about something other than the CIA." b. "It sounds like you're concerned about your privacy." c. "The CIA is prohibited from operating in health care facilities." d. "You have lost touch with reality, which is a symptom of your illness."

b. "It sounds like you're concerned about your privacy."

A woman who is 39 weeks pregnant expresses fear about her impending labor and how she will manage. What is the nurse's ideal response? a. "Don't worry about it. You'll do fine." b. "It's normal to be anxious about labor. Let's discuss what makes you afraid." c. "Labor is scary to think about, but the actual experience isn't." d. "You can have an epidural. You won't feel anything."

b. "It's normal to be anxious about labor. Let's discuss what makes you afraid."

A patient diagnosed with major depressive disorder tells the nurse, "Bad things that happen are always my fault." Which response by the nurse will best assist the patient to reframe this overgeneralization? a. "I really doubt that one person can be blamed for all the bad things that happen." b. "Let's look at one bad thing that happened to see if another explanation exists." c. "You are being extremely hard on yourself. Try to have a positive focus." d. "Are you saying that you don't have any good things happen?"

b. "Let's look at one bad thing that happened to see if another explanation exists."

Which statement by a newly delivered woman indicates that she knows what to expect regarding her menstrual activity after childbirth? a. "My first menstrual cycle will be lighter than normal and then will get heavier every month thereafter." b. "My first menstrual cycle will be heavier than normal and will return to my prepregnant volume within three or four cycles." c. "I will not have a menstrual cycle for 6 months after childbirth." d. "My first menstrual cycle will be heavier than normal and then will be light for several months after."

b. "My first menstrual cycle will be heavier than normal and will return to my prepregnant volume within three or four cycles."

While interviewing a 31-year-old woman before her routine gynecologic examination, the nurse collects data about the client's recent menstrual cycles. Which statement by the client should prompt the nurse to collect further information? a. "My menstrual flow lasts 5 to 6 days." b. "My flow is very heavy." c. "I have had a small amount of spotting midway between my periods for the past 2 months." d. "The length of my menstrual cycles varies from 26 to 29 days."

b. "My flow is very heavy."

A first-time mother is concerned about the type of medications she will receive during labor. The client is in a fair amount of pain and is nauseated. In addition, she appears to be very anxious. The nurse explains that opioid analgesics are often used along with sedatives. How should the nurse phrase the rationale for this medication combination? a. "The two medications, together, reduce complications." b. "Sedatives enhance the effect of the pain medication." c. "The two medications work better together, enabling you to sleep until you have the baby." d. "This is what your physician has ordered for you."

b. "Sedatives enhance the effect of the pain medication."

Which client is at greatest risk for early postpartum hemorrhage (PPH)? a. Primiparous woman (G 2, P 1-0-0-1) being prepared for an emergency cesarean birth for fetal distress b. Woman with severe preeclampsia on magnesium sulfate whose labor is being induced c. Multiparous woman (G 3, P 2-0-0-2) with an 8-hour labor d. Primigravida in spontaneous labor with preterm twins

b. Woman with severe preeclampsia on magnesium sulfate whose labor is being induced

A new client and her partner arrive on the labor, delivery, recovery, and postpartum (LDRP) unit for the birth of their first child. The nurse applies the electronic fetal monitor (EFM) to the woman. Her partner asks you to explain what is printing on the graph, referring to the EFM strip. He wants to know what the baby's heart rate should be. What is the nurse's best response? a. "Don't worry about that machine; that's my job." b. "The baby's heart rate will fluctuate in response to what is happening during labor." c. "The top line graphs the baby's heart rate, and the bottom line lets me know how strong the contractions are." d. "Your physician will explain all of that later."

b. "The baby's heart rate will fluctuate in response to what is happening during labor."

A perinatal nurse is giving discharge instructions to a woman, status post-suction, and curettage secondary to a hydatidiform mole. The woman asks why she must take oral contraceptives for the next 12 months. What is the best response by the nurse? a. "If you get pregnant within 1 year, the chance of a successful pregnancy is very small. Therefore, if you desire a future pregnancy, it would be better for you to use the most reliable method of contraception available." b. "The major risk to you after a molar pregnancy is a type of cancer that can be diagnosed only by measuring the same hormone that your body produces during pregnancy. If you were to get pregnant, then it would make the diagnosis of this cancer more difficult." c. "If you can avoid a pregnancy for the next year, the chance of developing a second molar pregnancy is rare. Therefore, to improve your chance of a successful pregnancy, not getting pregnant at this time is best." d. "Oral contraceptives are the only form of birth control that will prevent a recurrence of a molar pregnancy."

b. "The major risk to you after a molar pregnancy is a type of cancer that can be diagnosed only by measuring the same hormone that your body produces during pregnancy. If you were to get pregnant, then it would make the diagnosis of this cancer more difficult."

A pregnant woman at 29 weeks of gestation has been diagnosed with preterm labor. Her labor is being controlled with tocolytic medications. She asks when she might be able to go home. Which response by the nurse is most accurate? a. "After the baby is born." b. "When we can stabilize your preterm labor and arrange home health visits." c. "Whenever your physician says that it is okay." d. "It depends on what kind of insurance coverage you have."

b. "When we can stabilize your preterm labor and arrange home health visits."

A patient diagnosed with major depressive disorder says, "No one cares about me anymore. I'm not worth anything." Today the patient is wearing a new shirt and has neat, clean hair. Which remark by the nurse supports building a positive self-esteem for this patient? a. "You look nice this morning." b. "You're wearing a new shirt." c. "I like the shirt you are wearing." d. "You must be feeling better today."

b. "You're wearing a new shirt."

Which client is most likely to experience strong and uncomfortable afterpains? a. A woman who experienced oligohydramnios b. A woman who is a gravida 4, para 4-0-0-4 c. A woman who is bottle-feeding her infant d. A woman whose infant weighed 5 pounds, 3 ounces

b. A woman who is a gravida 4, para 4-0-0-4

Nurses can help their clients by keeping them informed about the distinctive stages of labor. Which description of the phases of the first stage of labor is accurate? a. Latent: Mild, regular contractions; no dilation; bloody show; duration of 2 to 4 hours b. Active: Moderate, regular contractions; 4- to 7-cm dilation; duration of 3 to 6 hours c. Lull: No contractions; dilation stable; duration of 20 to 60 minutes d. Transition: Very strong but irregular contractions; 8- to 10-cm dilation; duration of 1 to 2 hours

b. Active: Moderate, regular contractions; 4- to 7-cm dilation; duration of 3 to 6 hours

Which description of the phases of the first stage of labor is most accurate? a. Latent: mild, regular contractions; no dilation; bloody show b. Active: moderate, regular contractions; 4 to 7 cm dilation c. Lull: no contractions; dilation stable d. Transition: very strong but irregular contractions; 8 to 10 cm dilation

b. Active: moderate, regular contractions; 4 to 7 cm dilation

During a psychiatric assessment, the nurse observes a patient's facial expression is without emotion. The patient says, "Life feels so hopeless to me. I've been feeling sad for several months." How will the nurse document the patient's affect and mood? a. Affect depressed; mood flat b. Affect flat; mood depressed c. Affect labile; mood euphoric d. Affect and mood are incongruent.

b. Affect flat; mood depressed

Which statement regarding the postpartum uterus is correct? a. At the end of the third stage of labor, the postpartum uterus weighs approximately 500 g. b. After 2 weeks postpartum, it should be abdominally nonpalpable. c. After 2 weeks postpartum, it weighs 100 g. d. Postpartum uterus returns to its pre-pregnancy size by 6 weeks postpartum.

b. After 2 weeks postpartum, it should be abdominally nonpalpable.

Ideally, when should prenatal care begin? a. Before the first missed menstrual period b. After the first missed menstrual period c. After the second missed menstrual period d. After the third missed menstrual period

b. After the first missed menstrual period

Which sign of a potential complication is the most important for the nurse to share with the client? a. Constipation b. Alteration in the pattern of fetal movement c. Heart palpitations d. Edema in the ankles and feet at the end of the day

b. Alteration in the pattern of fetal movement

An adolescent girl tells the nurse that she is very suicidal. The nurse asks her if she has a specific plan. How should asking about a specific plan be viewed? a. Not a critical part of the assessment b. An appropriate part of the assessment c. Suggesting that adolescent needs a plan d. Encouraging adolescent to devise a plan

b. An appropriate part of the assessment

Which statement concerning the third stage of labor is correct? a. The placenta eventually detaches itself from a flaccid uterus. b. An expectant or active approach to managing this stage of labor reduces the risk of complications. c. It is important that the dark, roughened maternal surface of the placenta appears before the shiny fetal surface. d. The major risk for women during the third stage is a rapid heart rate.

b. An expectant or active approach to managing this stage of labor reduces the risk of complications.

Which finding on a prenatal visit at 10 weeks of gestation might suggest a hydatidiform mole? a. Complaint of frequent mild nausea b. Blood pressure of 120/80 mm Hg c. Fundal height measurement of 18 cm d. History of bright red spotting for 1 day, weeks ago

c. Fundal height measurement of 18 cm

A woman gave birth 48 hours ago to a healthy infant girl. She has decided to bottle feed. During the assessment, the nurse notices that both breasts are swollen, warm, and tender on palpation. Which guidance should the nurse provide to the client at this time? a. Run warm water on her breasts during a shower. b. Apply ice to the breasts for comfort. c. Express small amounts of milk from the breasts to relieve the pressure. d. Wearing a loose-fitting bra to prevent nipple irritation.

b. Apply ice to the breasts for comfort.

The breast-feeding mother should be taught to expect which changes to the condition of the breasts? (Select all that apply.) a. Breast tenderness is likely to persist for approximately 1 week after the start of lactation. b. As lactation is established, a mass may form that can be distinguished from cancer by its positional shift from day to day. c. In nonlactating mothers, colostrum is present for the first few days after childbirth. d. If suckling is never begun or is discontinued, then lactation ceases within a few days to a week. e. Little change occurs to the breasts in the first 48 hours.

b. As lactation is established, a mass may form that can be distinguished from cancer by its positional shift from day to day. c. In nonlactating mothers, colostrum is present for the first few days after childbirth. d. If suckling is never begun or is discontinued, then lactation ceases within a few days to a week.

Which nursing intervention is paramount when providing care to a client with preterm labor who has received terbutaline? a. Assess deep tendon reflexes (DTRs). b. Assess for dyspnea and crackles. c. Assess for bradycardia. d. Assess for hypoglycemia.

b. Assess for dyspnea and crackles.

Which key point is important for the nurse to understand regarding the perinatal continuum of care? a. Begins with conception and ends with the birth b. Begins with family planning and continues until the infant is 1 year old c. Begins with prenatal care and continues until the newborn is 24 weeks old d. Refers to home care only

b. Begins with family planning and continues until the infant is 1 year old

Which key factors play the most powerful role in the behaviors of individuals and families? a. Rituals and customs b. Beliefs and values c. Boundaries and channels d. Socialization processes

b. Beliefs and values

Which statement related to fetal positioning during labor is correct and important for the nurse to understand? a. Position is a measure of the degree of descent of the presenting part of the fetus through the birth canal. b. Birth is imminent when the presenting part is at +4 to +5 cm below the spine. c. The largest transverse diameter of the presenting part is the suboccipitobregmatic diameter. d. Engagement is the term used to describe the beginning of labor.

b. Birth is imminent when the presenting part is at +4 to +5 cm below the spine.

A tiered system of categorizing fetal heart rate (FHR) has been recommended by professional organizations. Nurses, midwives, and physicians who care for women in labor must have a working knowledge of fetal monitoring standards and understand the significance of each category. What is the correct nomenclature for these categories? (Select all that apply.) a. Reassuring b. Category I c. Category II d. Nonreassuring e. Category III

b. Category I c. Category II e. Category III

The nurse has received a report regarding a client in labor. The woman's last vaginal examination was recorded as 3 cm, 30%, and -2. What is the nurse's interpretation of this assessment? a. Cervix is effaced 3 cm and dilated 30%; the presenting part is 2 cm above the ischial spines. b. Cervix is dilated 3 cm and effaced 30%; the presenting part is 2 cm above the ischial spines. c. Cervix is effaced 3 cm and dilated 30%; the presenting part is 2 cm below the ischial spines. d. Cervix is dilated 3 cm and effaced 30%; the presenting part is 2 cm below the ischial spines.

b. Cervix is dilated 3 cm and effaced 30%; the presenting part is 2 cm above the ischial spines.

An older adult has experienced severe depression for many years and is unable to tolerate most antidepressant medications due to adverse effects of the medications. He is scheduled for electroconvulsive therapy (ECT) as a treatment for his depression. What teaching should the nurse give the patient regarding this treatment? a. There are no special preparations needed before this treatment. b. Common side effects include headache and short-term memory loss. c. One treatment will be needed to cure the depression. d. This treatment will leave you unconscious for several hours.

b. Common side effects include headache and short-term memory loss.

Which maternal condition always necessitates delivery by cesarean birth? a. Marginal placenta previa b. Complete placenta previa c. Ectopic pregnancy d. Eclampsia

b. Complete placenta previa

During the first trimester, which of the following changes regarding her sexual drive should a client be taught to expect? a. Increased sexual drive, because of enlarging breasts b. Decreased sexual drive, because of nausea and fatigue c. No change in her sexual drive d. Increased sexual drive, because of increased levels of female hormones

b. Decreased sexual drive, because of nausea and fatigue

When caring for a newly delivered woman, what is the best measure for the nurse to implement to prevent abdominal distention after a cesarean birth? a. Rectal suppositories b. Early and frequent ambulation c. Tightening and relaxing abdominal muscles d. Carbonated beverages

b. Early and frequent ambulation

A 20-year-old client is interested in protection from the human papilloma virus (HPV) since she may become sexually active. Which response from the nurse is the most accurate? a. You are too old to receive an HPV vaccine. b. Either Gardasil or Cervarix can provide protection. c. You will need to have three injections over a span of 1 year. d. The most common side effect of the vaccine is itching at the injection site.

b. Either Gardasil or Cervarix can provide protection. c. You will need to have three injections over a span of 1 year.

To assist a client in managing the symptoms of Premenstrual Syndrome (PMS), what intervention should the nurse recommend based on current evidence? a. Adhering to a diet with more body-building and energy foods, such as carbohydrates b. Engaging in herbal therapies, yoga, and massage therapy c. Seeking antidepressant therapy for symptom control d. Avoiding the use of natural diuretics

b. Engaging in herbal therapies, yoga, and massage therapy

What factor is most likely to increase the likelihood that an adolescent will misuse alcohol? a. Female gender b. Regular school attendance c. Rural environment d. Unconventional behavior

d. Unconventional behavior

A female infertility patient is found to be hypoestrogenic at the preconceptual clinic visit. She asks the nurse why she has never been able to get pregnant. Which response is best? a. Circulating estrogen contributes to secondary sex characteristics. b. Estrogen deficiency prevents the ovum from reaching the uterus and may be a factor in infertility. c. Hyperestrogen may be preventing the zona pellucida from forming an ovum protective layer. d. The corona radiata is preventing fertilization of the ovum.

b. Estrogen deficiency prevents the ovum from reaching the uterus and may be a factor in infertility.

A woman who is gravida 3 para 2 arrives on the intrapartum unit. What is the most important nursing assessment initially? a. Contraction pattern, amount of discomfort, and pregnancy history b. Fetal heart rate, maternal vital signs, and the woman's nearness to birth c. Identification of ruptured membranes, woman's gravida and para, and her support person d. Last food intake, when labor began, and cultural practices the couple desires

b. Fetal heart rate, maternal vital signs, and the woman's nearness to birth

What is a major physical risk for young adolescents during pregnancy? a. Osteoporosis frequently develops. b. Fetopelvic disproportion is a common problem. c. Delivery is usually precipitous in this age group. d. Pregnancy will adversely affect the adolescents development.

b. Fetopelvic disproportion is a common problem.

Through a vaginal examination, the nurse determines that a woman is 4 cm dilated. The external fetal monitor shows uterine contractions every to 4 minutes. The nurse reports this as what stage of labor? a. First stage, latent phase b. First stage, active phase c. First stage, transition phase d. Second stage, latent phase

b. First stage, active phase

What statement is true concerning adolescent suicide? a. A sense of hopelessness and despair is a normal part of adolescence. b. Gay and lesbian adolescents are at a particularly high risk for suicide. c. Problem-solving skills are of limited value to the suicidal adolescent. d. Previous suicide attempts are not an indication for completed suicides.

b. Gay and lesbian adolescents are at a particularly high risk for suicide.

What is the usual presenting symptom for testicular cancer? a. Solid, painful mass b. Hard, painless mass c. Scrotal swelling and pain d. Epididymis easily palpated

b. Hard, painless mass

What is the primary difference between hospital care and home health care? a. Home care is routinely and continuously delivered by professional staff. b. Home care is delivered on an intermittent basis by professional staff. c. Home care is delivered for emergency conditions. d. Home care is not available 24 hours a day.

b. Home care is delivered on an intermittent basis by professional staff.

A 3-year-old girl's mother is 6 months pregnant. What concern is this child most likely to verbalize? a. How the baby will get out? b. How will the baby eat? c. Will you die having the baby? d. What color eyes will the baby have?

b. How will the baby eat?

A client with a history of bipolar disorder is called by the postpartum support nurse for follow-up. Which symptoms would reassure the nurse that the client is not experiencing a manic episode? a. Psychomotor agitation and rapid speech patterns b. Increased appetite and sleeping 8 hours a night c. Hyperactivity and distractibility d. Pressured speech and grandiosity

b. Increased appetite and sleeping 8 hours a night

In addition to eye contact, other early sensual contacts between the infant and mother involve sound and smell. What other statement regarding the senses is correct? a. High-pitched voices irritate newborns. b. Infants can learn to distinguish their mother's voice from others soon after birth. c. All babies in the hospital smell alike. d. Mother's breast milk has no distinctive odor.

b. Infants can learn to distinguish their mother's voice from others soon after birth.

According to professional standards which action cannot be performed by the non-anesthetist registered nurse who is caring for a woman with epidural anesthesia? a. Monitoring the status of the woman and fetus b. Initiating epidural anesthesia c. Replacing empty infusion bags with the same medication and concentrate d. Stopping the infusion, and initiating emergency measures

b. Initiating epidural anesthesia

In contrast to placenta previa, what is the most prevalent clinical manifestation of abruptio placentae? a. Bleeding b. Intense abdominal pain c. Uterine activity d. Cramping

b. Intense abdominal pain

A client has recently been diagnosed with stage III endometrial cancer and asks the nurse for an explanation. What response by the nurse is correct about the staging of the cancer? a. The cancer has spread to the mucosa of the bowel and bladder. b. It has reached the vagina or lymph nodes. c. The cancer now involves the cervix. d. It is contained in the endometrium of the cervix.

b. It has reached the vagina or lymph nodes.

A number of methods can be used for inducing labor. Which cervical ripening method falls under the category of mechanical or physical? a. Prostaglandins are used to soften and thin the cervix. b. Labor can sometimes be induced with balloon catheters or laminaria tents. c. Oxytocin is less expensive and more effective than prostaglandins but creates greater health risks. d. Amniotomy can be used to make the cervix more favorable for labor.

b. Labor can sometimes be induced with balloon catheters or laminaria tents.

What would a steady trickle of bright red blood from the vagina in the presence of a firm fundus suggest to the nurse? a. Uterine atony b. Lacerations of the genital tract c. Perineal hematoma d. Infection of the uterus

b. Lacerations of the genital tract

Which fetal heart rate (FHR) decelerations would require the nurse to change the maternal position? (Select all that apply.) a. Early decelerations b. Late decelerations c. Variable decelerations d. Moderate decelerations e. Prolonged decelerations

b. Late decelerations c. Variable decelerations e. Prolonged decelerations

Which characteristic correctly matches the type of deceleration with its likely cause? a. Early deceleration—umbilical cord compression b. Late deceleration—uteroplacental insufficiency c. Variable deceleration—head compression d. Prolonged deceleration—unknown cause

b. Late deceleration—uteroplacental insufficiency

A woman who has a history of sexual abuse may have several traumatic memories triggered during labor. She may fight the labor process and react with pain or anger. The nurse can implement appropriate care measures to help her client view the childbirth experience in a positive manner. Which intervention is key for the nurse to use while providing care? a. Tell the client to relax and that it won't hurt much. b. Limit the number of procedures that invade her body. c. Reassure the client that, as the nurse, you know what is best. d. Allow unlimited care providers to be with the client.

b. Limit the number of procedures that invade her body.

A nurse taught a patient about a tyramine-restricted diet. Which menu selection would the nurse approve? a. Macaroni and cheese, hot dogs, banana bread, caffeinated coffee b. Mashed potatoes, ground beef patty, corn, green beans, apple pie c. Avocado salad, ham, creamed potatoes, asparagus, chocolate cake d. Noodles with cheddar cheese sauce, smoked sausage, lettuce salad, yeast rolls

b. Mashed potatoes, ground beef patty, corn, green beans, apple pie

Which statement is the best rationale for assessing the maternal vital signs between uterine contractions? a. During a contraction, assessing the fetal heart rate is the priority. b. Maternal circulating blood volume temporarily increases during contractions. c. Maternal blood flow to the heart is reduced during contractions. d. Vital signs taken during contractions are not accurate.

b. Maternal circulating blood volume temporarily increases during contractions.

What is a maternal indication for the use of forceps-assisted birth? a. Wide pelvic outlet b. Maternal exhaustion c. History of rapid deliveries d. Failure to progress past station 0

b. Maternal exhaustion

The use of methamphetamine (meth) has been described as a significant drug problem in the United States. The nurse who provides care to this client population should be cognizant of what regarding methamphetamine use? a. Methamphetamines are similar to opiates. b. Methamphetamines are stimulants with vasoconstrictive characteristics. c. Methamphetamines should not be discontinued during pregnancy. d. Methamphetamines are associated with a low rate of relapse.

b. Methamphetamines are stimulants with vasoconstrictive characteristics.

What is the rationale for the use of a blood patch after spinal anesthesia? a. Preventing related hypotension b. Minimizing the risk of a spinal headache c. Eliminating neonatal respiratory depression d. Limiting the loss of movement

b. Minimizing the risk of a spinal headache

What is the correct terminology for an abortion in which the fetus dies but is retained within the uterus? a. Inevitable abortion b. Missed abortion c. Incomplete abortion d. Threatened abortion

b. Missed abortion

The nurse is caring for an adolescent brought to the hospital with acute drug toxicity. Cocaine is believed to be the drug involved. Data collection by the nurse should include what information? a. Drugs actual content b. Mode of administration c. Adolescents level of interest in rehabilitation d. Function the drug plays in the adolescents life

b. Mode of administration

What type of cultural concern is the most likely deterrent to many women seeking prenatal care? a. Religion b. Modesty c. Ignorance d. Belief that physicians are evil

b. Modesty

What is the correct term describing the slight overlapping of cranial bones or shaping of the fetal head during labor? a. Lightening b. Molding c. Ferguson reflex d. Valsalva maneuver

b. Molding

During labor a fetus displays an average fetal heart rate (FHR) of 135 beats per minute over a 10-minute period. Which statement best describes the status of this fetus? a. Bradycardia b. Normal baseline heart rate c. Tachycardia d. Hypoxia

b. Normal baseline heart rate

Which statement about the development of cultural competence is inaccurate? a. Local health care workers and community advocates can help extend health care to underserved populations. b. Nursing care is delivered in the context of the client's culture but not in the context of the nurse's culture. c. Nurses must develop an awareness of and a sensitivity to various cultures. d. Culture's economic, religious, and political structures influence practices that affect childbearing.

b. Nursing care is delivered in the context of the client's culture but not in the context of the nurse's culture.

An adolescent girl is brought to the hospital emergency department by her parents after being raped. The girl is calm and controlled throughout the interview and examination. The nurse should recognize this behavior is what? a. A sign that a rape has not actually occurred b. One of a variety of behaviors normally seen in rape victims c. Indicative of a higher than usual level of maturity in the adolescent d. Suggestive that the adolescent had severe emotional problems before the rape occurred

b. One of a variety of behaviors normally seen in rape victims

The nurse is teaching a class on obesity prevention to parents in the community. What is a contributing factor to childhood obesity? a. Birth weight b. Parental overweight c. Age at the onset of puberty d. Asian ethnic background

b. Parental overweight

In follow-up appointments or visits with parents and their new baby, it may be useful if the nurse can identify parental behaviors that can either facilitate or inhibit attachment. Which is a facilitating behavior? a. Parents have difficulty naming the infant. b. Parents hover around the infant, directing attention to and pointing at the infant. c. Parents make no effort to interpret the actions or needs of the infant. d. Parents do not move from fingertip touch to palmar contact and holding.

b. Parents hover around the infant, directing attention to and pointing at the infant.

The postpartum nurse should be cognizant of what about the adaptation of other family members (primarily siblings and grandparents) to the newborn? a. Sibling rivalry cannot be dismissed as overblown psychobabble; negative feelings and behaviors can take a long time to blow over. b. Participation in preparation classes helps both siblings and grandparents. c. In the United States, paternal and maternal grandparents consider themselves of equal importance and status. d. Since 1990, the number of grandparents providing permanent care to their grandchildren has been declining.

b. Participation in preparation classes helps both siblings and grandparents.

The number of routine laboratory tests during follow-up visits is limited; however, those that are performed are essential. Which statements regarding group B Streptococcus (GBS) testing are correct? (Select all that apply.) a. Performed between 32 and 34 weeks of gestation. b. Performed between 35 and 37 weeks of gestation. c. All women should be tested. d. Only women planning a vaginal birth should be tested. e. Women with a history of GBS should be retested.

b. Performed between 35 and 37 weeks of gestation. c. All women should be tested. e. Women with a history of GBS should be retested.

What is the primary nursing responsibility when caring for a client who is experiencing an obstetric hemorrhage associated with uterine atony? a. Establishing venous access b. Performing fundal massage c. Preparing the woman for surgical intervention d. Catheterizing the bladder

b. Performing fundal massage

Maternal hypotension is a potential side effect of regional anesthesia and analgesia. What nursing interventions could the nurse use to increase the client's blood pressure? (Select all that apply.) a. Place the woman in a supine position. b. Place the woman in a lateral position. c. Increase IV fluids. d. Administer oxygen. e. Perform a vaginal examination.

b. Place the woman in a lateral position. c. Increase IV fluids. d. Administer oxygen.

In the United States, the en face position is preferred immediately after birth. Which actions by the nurse can facilitate this process? (Select all that apply.) a. Washing both the infant's face and the mother's face b. Placing the infant on the mother's abdomen or breast with their heads on the same plane c. Dimming the lights d. Delaying the instillation of prophylactic antibiotic ointment in the infant's eyes e. Placing the infant in the grandmother's arms

b. Placing the infant on the mother's abdomen or breast with their heads on the same plane c. Dimming the lights d. Delaying the instillation of prophylactic antibiotic ointment in the infant's eyes

Which laboratory marker is indicative of disseminated intravascular coagulation (DIC)? a. Bleeding time of 10 minutes b. Presence of fibrin split products c. Thrombocytopenia d. Hypofibrinogenemia

b. Presence of fibrin split products

What is the correct terminology for the nerve block that provides anesthesia to the lower vagina and perineum? a. Epidural b. Pudendal c. Local d. Spinal block

b. Pudendal

A 15-year-old girl tells the school nurse that she has not started to menstruate yet. Onset of secondary sexual characteristics was about 2 1/2 years ago. The nurse should take which action? a. Explain that this is not unusual. b. Refer the adolescent for an evaluation. c. Make an assumption that the adolescent is pregnant. d. Suggest that the adolescent stop exercising until menarche occurs.

b. Refer the adolescent for an evaluation.

A client is admitted to the emergency department with toxic shock syndrome. Which action by the nurse is the most important? a. Administer IV fluids to maintain fluid and electrolyte balance. b. Remove the tampon as the source of infection. c. Collect a blood specimen for culture and sensitivity. d. Transfuse the client to manage low blood count.

b. Remove the tampon as the source of infection.

The nurse should be cognizant of which postpartum physiologic alteration? a. Cardiac output, pulse rate, and stroke volume all return to prepregnancy normal values within 48 hours of childbirth. b. Respiratory function returns to nonpregnant levels by 6 to 8 weeks after childbirth. c. Lowered white blood cell count after pregnancy can lead to false-positive results on tests for infections. d. Hypercoagulable state protects the new mother from thromboembolism, especially after a cesarean birth.

b. Respiratory function returns to nonpregnant levels by 6 to 8 weeks after childbirth.

What do nursing responsibilities regarding weight gain for an adolescent with anorexia nervosa include? a. Administer tube feedings until target weight is achieved. b. Restore body weight to within 10% of the adolescents ideal weight. c. Encourage continuation of strenuous exercise as long as adolescent is not losing weight. d. Facilitate as rapid a weight gain as possible with a high-calorie diet.

b. Restore body weight to within 10% of the adolescents ideal weight.

A patient diagnosed with major depressive disorder repeatedly tells staff, "I have cancer. It's my punishment for being a bad person." Diagnostic tests reveal no cancer. Select the priority nursing diagnosis. a. Powerlessness b. Risk for suicide c. Stress overload d. Spiritual distress

b. Risk for suicide

Anxiety is commonly associated with pain during labor. Which statement regarding anxiety is correct? a. Even mild anxiety must be treated. b. Severe anxiety increases tension, increases pain, and then, in turn, increases fearand anxiety, and so on. c. Anxiety may increase the perception of pain, but it does not affect the mechanism of labor. d. Women who have had a painful labor will have learned from the experience and have less anxiety the second time because of increased familiarity.

b. Severe anxiety increases tension, increases pain, and then, in turn, increases fearand anxiety, and so on.

The nurse observes several interactions between a postpartum woman and her new son. What behavior, if exhibited by this woman, would the nurse identify as a possible maladaptive behavior regarding parent-infant attachment? a. The postpartum woman talks and coos to her son. b. She seldom makes eye contact with her son. c. The mother cuddles her son close to her. d. She tells visitors how well her son is feeding.

b. She seldom makes eye contact with her son.

The nurse observes that a first-time mother appears to ignore her newborn. Which strategy should the nurse use to facilitate mother-infant attachment? a. Tell the mother she must pay attention to her infant. b. Show the mother how the infant initiates interaction and attends to her. c. Demonstrate for the mother different positions for holding her infant while feeding. d. Arrange for the mother to watch a video on parent-infant interaction.

b. Show the mother how the infant initiates interaction and attends to her.

What is the most significant factor in distinguishing those who commit suicide from those who make suicidal attempts or threats? a. Level of stress b. Social isolation c. Degree of depression d. Desire to punish others

b. Social isolation

During an inpatient psychiatric hospitalization, what is the most important nursing intervention? a. Contacting the client's significant other b. Supervising and guiding visits with her infant c. Allowing no contact with anyone who annoys her d. Having the infant with the mother the majority of the time

b. Supervising and guiding visits with her infant

According to the National Institute of Child Health and Human Development (NICHD) Three-Tier System of FHR Classification, category III tracings include all FHR tracings not categorized as category I or II. Which characteristics of the fetal heart rate (FHR) belong in category III? (Select all that apply.) a. Baseline rate of 110 to 160 beats per minute b. Tachycardia c. Absent baseline variability not accompanied by recurrent decelerations d. Variable decelerations with other characteristics such as shoulders or overshoots e. Absent baseline variability with recurrent variable decelerations f. Bradycardia

b. Tachycardia d. Variable decelerations with other characteristics such as shoulders or overshoots e. Absent baseline variability with recurrent variable decelerations f. Bradycardia

During the physical examination of a client beginning prenatal care, which initial action is most important for the nurse to perform? a. Only women who show physical signs or meet the sociologic profile should be assessed for physical abuse. b. The client should empty her bladder before the pelvic examination. c. The distribution, amount, and quality of body hair are of no particular importance. d. The size of the uterus is discounted in the initial examination because it will be increasing in size during the second trimester.

b. The client should empty her bladder before the pelvic examination.

During the first interview with a parent whose child died in a car accident, the nurse feels empathic and reaches out to take the patient's hand. Select the correct analysis of the nurse's behavior. a. It shows empathy and compassion. It will encourage the patient to continue to express feelings. b. The gesture is premature. The patient's cultural and individual interpretation of touch is unknown. c. The patient will perceive the gesture as intrusive and overstepping boundaries. d. The action is inappropriate. Psychiatric patients should not be touched.

b. The gesture is premature. The patient's cultural and individual interpretation of touch is unknown.

A pregnant woman is being discharged from the hospital after the placement of a cervical cerclage because of a history of recurrent pregnancy loss, secondary to an incompetent cervix. Which information regarding postprocedural care should the nurse emphasize in the discharge teaching? a. Any vaginal discharge should be immediately reported to her health care provider. b. The presence of any contractions, rupture of membranes (ROM), or severe perineal pressure should be reported. c. The client will need to arrange for care at home, because her activity level will be restricted. d. The client will be scheduled for a cesarean birth.

b. The presence of any contractions, rupture of membranes (ROM), or severe perineal pressure should be reported.

The nurse is doing preoperative teaching for a client who is scheduled for removal of cervical polyps in the office. Which statement by the client indicates a correct understanding of the procedure? a. I hope that I do not have cancer of the cervix. b. There should be little or no discomfort during the procedure. c. There may be a lot of bleeding after the polyp is removed. d. This may prevent me from having any more children.

b. There should be little or no discomfort during the procedure.

The client is instructed to place her thumb and forefinger on the areola and gently press inward. What is the purpose of this exercise? a. To check the sensitivity of the nipples b. To determine whether the nipple is everted or inverted c. To calculate the adipose buildup in the abdomen d. To see whether the fetus has become inactive

b. To determine whether the nipple is everted or inverted

After an emergency birth, the nurse encourages the woman to breastfeed her newborn. What is the primary purpose of this activity? a. To facilitate maternal-newborn interaction b. To stimulate the uterus to contract c. To prevent neonatal hypoglycemia d. To initiate the lactation cycle

b. To stimulate the uterus to contract

What is the rationale for the administration of an oxytocic after expulsion of the placenta? a. To relieve pain b. To stimulate uterine contraction c. To prevent infection d. To facilitate rest and relaxation

b. To stimulate uterine contraction

What is the primary difference between the labor of a nullipara and that of a multipara? a. Amount of cervical dilation b. Total duration of labor c. Level of pain experienced d. Sequence of labor mechanisms

b. Total duration of labor

A 26-year-old pregnant woman, gravida 2, para 1-0-0-1, is 28 weeks pregnant when she experiences bright red, painless vaginal bleeding. On her arrival at the hospital, which diagnostic procedure will the client most likely have performed? a. Amniocentesis for fetal lung maturity b. Transvaginal ultrasound for placental location c. Contraction stress test (CST) d. Internal fetal monitoring

b. Transvaginal ultrasound for placental location

Which statement regarding multifetal pregnancy is incorrect? a. The expectant mother often develops anemia because the fetuses have a greater demand for iron. b. Twin pregnancies come to term with the same frequency as single pregnancies. c. The mother should be counseled to increase her nutritional intake and gain more weight. d. Backache and varicose veins often are more pronounced with a multifetal pregnancy.

b. Twin pregnancies come to term with the same frequency as single pregnancies.

What is the most likely cause for variable fetal heart rate (FHR) decelerations? a. Altered fetal cerebral blood flow b. Umbilical cord compression c. Uteroplacental insufficiency d. Fetal hypoxemia

b. Umbilical cord compression

When the services of an interpreter are needed, which is the most important factor for the nurse to consider? a. Using a family member who is fluent in both languages b. Using an interpreter who is certified, and documenting the person's name in the nursing notes c. Directing questions only to the interpreter d. Using an interpreter only in an emergency

b. Using an interpreter who is certified, and documenting the person's name in the nursing notes

Which description of postpartum restoration or healing times is accurate? a. The cervix shortens, becomes firm, and returns to form within a month postpartum. b. Vaginal rugae reappear by 3 weeks postpartum. c. Most episiotomies heal within a week. d. Hemorrhoids usually decrease in size within 2 weeks of childbirth.

b. Vaginal rugae reappear by 3 weeks postpartum.

Under which circumstance would it be unnecessary for the nurse to perform a vaginal examination? a. Admission to the hospital at the start of labor b. When accelerations of the fetal heart rate (FHR) are noted c. On maternal perception of perineal pressure or the urge to bear down d. When membranes rupture

b. When accelerations of the fetal heart rate (FHR) are noted

What information should the nurse understand fully regarding rubella and Rh status? a. Breastfeeding mothers cannot be vaccinated with the live attenuated rubella virus. b. Women should be warned that the rubella vaccination is teratogenic and that they must avoid pregnancy for at least 1 month after vaccination. c. Rh immunoglobulin is safely administered intravenously because it cannot harm a nursing infant. d. Rh immunoglobulin boosts the immune system and thereby enhances the effectiveness of vaccinations.

b. Women should be warned that the rubella vaccination is teratogenic and that they must avoid pregnancy for at least 1 month after vaccination.

A client is scheduled to start external beam radiation therapy (EBRT) for her endometrial cancer. Which teaching by the nurse is accurate? (Select all that apply.) a. You will need to be hospitalized during this therapy. b. Your skin needs to be inspected daily for any breakdown. c. It is not wise to stay out in the sun for long periods of time. d. The perineal area may become damaged with the radiation. e. The technician applies new site markings before each treatment.

b. Your skin needs to be inspected daily for any breakdown. c. It is not wise to stay out in the sun for long periods of time. d. The perineal area may become damaged with the radiation.

A patient says to the nurse, "My life doesn't have any happiness in it anymore. I once enjoyed holidays, but now they're just another day." The nurse documents this report as an example of a. dysthymia. b. anhedonia. c. euphoria. d. anergia.

b. anhedonia.

Priority interventions for a patient diagnosed with major depressive disorder and feelings of worthlessness should include a. distracting the patient from self-absorption. b. careful unobtrusive observation around the clock. c. allowing the patient to spend long periods alone in meditation. d. opportunities to assume a leadership role in the therapeutic milieu.

b. careful unobtrusive observation around the clock.

The patient says, "My marriage is just great. My spouse and I always agree." The nurse observes the patient's foot moving continuously as the patient twirls a shirt button. The conclusion the nurse can draw is that the patient's communication is a. clear. b. distorted. (mixed) c. incongruous. d. inadequate.

b. distorted. (mixed)

A nurse instructs a patient taking a medication that inhibits the action of monoamine oxidase (MAO) to avoid certain foods and drugs because of the risk of a. hypotensive shock. b. hypertensive crisis. c. cardiac dysrhythmia. d. cardiogenic shock.

b. hypertensive crisis.

A black patient says to a white nurse, "There's no sense talking about how I feel. You wouldn't understand because you live in a white world." The nurse's best action would be to a. explain, "Yes, I do understand. Everyone goes through the same experiences." b. say, "Please give an example of something you think I wouldn't understand." c. reassure the patient that nurses interact with people from all cultures. d. change the subject to one that is less emotionally disturbing.

b. say, "Please give an example of something you think I wouldn't understand."

A patient discloses several concerns and associated feelings. If the nurse wants to seek clarification, which comment would be appropriate? a. "What are the common elements here?" b. "Tell me again about your experiences." c. "Am I correct in understanding that." d. "Tell me everything from the beginning."

c. "Am I correct in understanding that."

The nurse is seeking clarification of a statement that was made by a patient. What is the best way for the nurse to seek clarification? a. "What are the common elements here?" b. "Tell me again about your experiences." c. "Am I correct in understanding that..." d. "Tell me everything from the beginning."

c. "Am I correct in understanding that..."

Which comments by a nurse demonstrate use of therapeutic communication techniques? (Select all that apply.) a. "Why do you think these events have happened to you?" b. "There are people with problems much worse than yours." c. "I'm glad you were able to tell me how you felt about your loss." d. "I noticed your hands trembling when you told me about your accident." e. "You look very nice today. I'm proud you took more time with your appearance."

c. "I'm glad you were able to tell me how you felt about your loss." d. "I noticed your hands trembling when you told me about your accident."

A patient has been prescribed a selective serotonin reuptake inhibitor (SSRI) antidepressant. After taking the new medication, the patient states, "This medication isn't working. I don't feel any different." What is the best response by the nurse? a. "I will call your care provider. Perhaps you need a different medication." b. "Don't worry. You can try taking it at a different time of day to help it work better." c. "It usually takes a few weeks for you to notice improvement from this medication." d. "Your life is much better now. You will feel better soon."

c. "It usually takes a few weeks for you to notice improvement from this medication."

A patient being treated for depression has taken sertraline daily for a year. The patient calls the clinic nurse and says, "I stopped taking my antidepressant 2 days ago. Now I am having nausea, nervous feelings, and I can't sleep." The nurse will advise the patient to: a. "Go to the nearest emergency department immediately." b. "Do not to be alarmed. Take two aspirin and drink plenty of fluids." c. "Take a dose of your antidepressant now and come to the clinic to see the health care provider." d. "Resume taking your antidepressants for 2 more weeks and then discontinue them again."

c. "Take a dose of your antidepressant now and come to the clinic to see the health care provider."

Which statement by the client will assist the nurse in determining whether she is in true labor as opposed to false labor? a. "I passed some thick, pink mucus when I urinated this morning." b. "My bag of waters just broke." c. "The contractions in my uterus are getting stronger and closer together." d. "My baby dropped, and I have to urinate more frequently now."

c. "The contractions in my uterus are getting stronger and closer together."

Which statement by the client would lead the nurse to believe that labor has been established? a. "I passed some thick, pink mucus when I urinated this morning." b. "My bag of waters just broke." c. "The contractions in my uterus are getting stronger and closer together." d. "My baby dropped, and I have to urinate more frequently now."

c. "The contractions in my uterus are getting stronger and closer together."

A patient tells the nurse, "I don't think I'll ever get out of here." Select the nurse's most therapeutic response. a. "Don't talk that way. Of course you will leave here!" b. "Keep up the good work, and you certainly will." c. "You don't think you're making progress?" d. "Everyone feels that way sometimes."

c. "You don't think you're making progress?"

A pregnant woman at 10 weeks of gestation jogs three or four times per week. She is concerned about the effect of the exercise on the fetus. Which guidance should the nurse provide? a. "You don't need to modify your exercising any time during your pregnancy." b. "Stop exercising because it will harm the fetus." c. "You may find that you need to modify your exercise to walking later in your pregnancy, around the seventh month." d. "Jogging is too hard on your joints; switch to walking now."

c. "You may find that you need to modify your exercise to walking later in your pregnancy, around the seventh month."

Changes in blood volume after childbirth depend on several factors such as blood loss during childbirth and the amount of extravascular water (physiologic edema) mobilized and excreted. What amount of blood loss does the postpartum nurse anticipate? (Select all that apply.) a. 100 ml b. 250 ml or less c. 300 to 500 ml d. 500 to 1000 ml e. 1500 ml or greater

c. 300 to 500 ml d. 500 to 1000 ml

Which information related to a prolonged deceleration is important for the labor nurse to understand? a. Prolonged decelerations present a continuing pattern of benign decelerations that do not require intervention. b. Prolonged decelerations constitute a baseline change when they last longer than 5 minutes. c. A disruption to the fetal oxygen supply causes prolonged decelerations. d. Prolonged decelerations require the customary fetal monitoring by the nurse.

c. A disruption to the fetal oxygen supply causes prolonged decelerations.

A nulliparous woman has just begun the latent phase of the second stage of her labor. The nurse should anticipate which behavior? a. A nulliparous woman will experience a strong urge to bear down. b. Perineal bulging will show. c. A nulliparous woman will remain quiet with her eyes closed between contractions. d. The amount of bright red bloody show will increase.

c. A nulliparous woman will remain quiet with her eyes closed between contractions.

The nurse is teaching a client who is undergoing brachytherapy about what to immediately report to her health care provider. Which signs and symptoms would be included in this teaching? (Select all that apply.) a. Constipation for 3 days b. Temperature of 99 F c. Abdominal pain d. Visible blood in the urine e. Heavy vaginal bleeding

c. Abdominal pain d. Visible blood in the urine e. Heavy vaginal bleeding

A female patient comes to the clinic after missing one menstrual period. She lives in a house beneath electrical power lines which is located near an oil field. She drinks two caffeinated beverages a day, is a daily beer drinker, and has not stopped eating sweets. She takes a multivitamin and exercises daily. She denies drug use. Which finding in the history has the greatest implication for this patient's plan of care? a. Electrical power lines are a potential hazard to the woman and her fetus. b. Living near an oil field may mean the water supply is polluted. c. Alcohol exposure should be avoided during pregnancy due to teratogenicity. d. Eating sweets may cause gestational diabetes or miscarriage.

c. Alcohol exposure should be avoided during pregnancy due to teratogenicity.

A woman in labor is breathing into a mouthpiece just before the start of her regular contractions. As she inhales, a valve opens, and gas is released. She continues to inhale the gas slowly and deeply until the contraction starts to subside. When the inhalation stops, the valve closes. Which statement regarding this procedure is correct? a. The application of nitrous oxide gas is not often used anymore. b. An inhalation of gas is likely to be used in the second stage of labor, not during the first stage. c. An application of nitrous oxide gas is administered for pain relief. d. The application of gas is a prelude to a cesarean birth.

c. An application of nitrous oxide gas is administered for pain relief.

Tretinoin (Retin-A) is a commonly used topical agent for the treatment of acne. What do nursing considerations with this drug include? a. Sun exposure increases effectiveness. b. Cosmetics with lanolin and petrolatum are preferred in acne. c. Applying of the medication occurs at least 20 to 30 minutes after washing. d. Erythema and peeling are indications of toxicity and need to be reported.

c. Applying of the medication occurs at least 20 to 30 minutes after washing.

The nurse is performing an initial assessment of a client in labor. What is the appropriate terminology for the relationship of the fetal body parts to one another? a. Lie b. Presentation c. Attitude d. Position

c. Attitude

Which statement related to cephalopelvic disproportion (CPD) is the least accurate? a. CPD can be related to either fetal size or fetal position. b. The fetus cannot be born vaginally. c. CPD can be accurately predicted. d. Causes of CPD may have maternal or fetal origins.

c. CPD can be accurately predicted.

A nurse hears a primiparous woman talking to her son and telling him that his chin is just like his dad's. This statement is most descriptive of which process? a. Mutuality b. Synchrony c. Claiming d. Reciprocity

c. Claiming

What goal is most important when caring for a child with anorexia nervosa (AN)? a. Limit fluid intake. b. Prevent depression. c. Correct malnutrition. d. Encourage weight gain.

c. Correct malnutrition.

A woman arrives for evaluation of signs and symptoms that include a missed period, adnexal fullness, tenderness, and dark red vaginal bleeding. On examination, the nurse notices an ecchymotic blueness around the woman's umbilicus. What does this finding indicate? a. Normal integumentary changes associated with pregnancy b. Turner sign associated with appendicitis c. Cullen sign associated with a ruptured ectopic pregnancy d. Chadwick sign associated with early pregnancy

c. Cullen sign associated with a ruptured ectopic pregnancy

A 55-year-old post-menopausal woman is assessed by the nurse with a history of dyspareunia, backache, pelvis pressure, urinary tract infections, and a frequent urinary urgency. Which condition does the nurse suspect? a. Ovarian cyst b. Rectocele c. Cystocele d. Fibroid

c. Cystocele

Which description of the phases of the second stage of labor is most accurate? a. Latent phase: Feeling sleepy; fetal station 2+ to 4+; duration of 30 to 45 minutes b. Active phase: Overwhelmingly strong contractions; Ferguson reflux activated; duration of 5 to 15 minutes c. Descent phase: Significant increase in contractions; Ferguson reflux activated; average duration varies d. Transitional phase: Woman "laboring down"; fetal station 0; duration of 15 minutes

c. Descent phase: Significant increase in contractions; Ferguson reflux activated; average duration varies

Which is the initial treatment for the client diagnosed with von Willebrand disease (vWD) who experiences a postpartum hemorrhage (PPH)? a. Cryoprecipitate b. Factor VIII and von Willebrand factor (vWf) c. Desmopressin d. Hemabate

c. Desmopressin

Where is the point of maximal intensity (PMI) of the fetal heart tone (FHR) located? a. Usually directly over the fetal abdomen b. In a vertex position, heard above the mother's umbilicus c. Directly over the fetal back d. In a breech position, heard below the mother's umbilicus

c. Directly over the fetal back

Which is the most accurate description of postpartum depression (PPD) without psychotic features? a. Postpartum baby blues requiring the woman to visit with a counselor or psychologist b. Condition that is more common among older Caucasian women because they have higher expectations c. Distinguishable by pervasive sadness along with mood swings d. Condition that disappears without outside help

c. Distinguishable by pervasive sadness along with mood swings

The uterine contractions of a woman early in the active phase of labor are assessed by an internal uterine pressure catheter (IUPC). The uterine contractions occur every 3 to 4 minutes and last an average of 55 to 60 seconds. They are becoming more regular and are moderate to strong. Based on this information, what would a prudent nurse do next? a. Immediately notify the woman's primary health care provider. b. Prepare to administer an oxytocic to stimulate uterine activity. c. Document the findings because they reflect the expected contraction pattern for the active phase of labor. d. Prepare the woman for the onset of the second stage of labor.

c. Document the findings because they reflect the expected contraction pattern for the active phase of labor.

Which consideration is essential for the nurse to understand regarding follow-up prenatal care visits? a. The interview portions become more intensive as the visits become more frequent over the course of the pregnancy. b. Monthly visits are scheduled for the first trimester, every 2 weeks for the second trimester, and weekly for the third trimester. c. During the abdominal examination, the nurse should be alert for supine hypotension. d. For pregnant women, a systolic BP of 130 mm Hg and a diastolic BP of 80 mm Hg is sufficient to be considered hypertensive.

c. During the abdominal examination, the nurse should be alert for supine hypotension.

A patient has been resistant to treatment with antidepressant therapy. The care provider prescribes a monoamine oxidase inhibitor (MAOI) medication. What teaching is critical for the nurse to give the patient? a. Serum blood levels must be regularly monitored to assess for toxicity. b. To prevent side effects, the medication should be administered as an intramuscular injection. c. Eating foods such as blue cheese or red wine will cause side effects. d. This medication class may only be used safely for a few days at a time.

c. Eating foods such as blue cheese or red wine will cause side effects.

Nurses should be cognizant of what regarding the mechanism of labor? a. Seven critical movements usually progress in a orderly sequence. b. Asynclitism is sometimes achieved by means of the Leopold's maneuver. c. Effects of the forces determining descent are modified by the shape of the woman's pelvis and the size of the fetal head. d. At birth, the baby is said to achieve "restitution"; that is, a return to the C-shape of the womb.

c. Effects of the forces determining descent are modified by the shape of the woman's pelvis and the size of the fetal head.

Which statement related to the condition of endometriosis accurately describes it's pathophysiology? a. Endometriosis is characterized by the presence and growth of endometrial tissue inside the uterus. b. It is found more often in African-American women than in Caucasian or Asian women. c. Endometriosis may worsen with repeated cycles or remain asymptomatic and disappear after menopause. d. It is unlikely to affect sexual intercourse or fertility.

c. Endometriosis may worsen with repeated cycles or remain asymptomatic and disappear after menopause.

Which action is correct when palpation is used to assess the characteristics and pattern of uterine contractions? a. Placing the hand on the abdomen below the umbilicus and palpating uterine tone with the fingertips b. Determining the frequency by timing from the end of one contraction to the end of the next contraction c. Evaluating the intensity by pressing the fingertips into the uterine fundus d. Assessing uterine contractions every 30 minutes throughout the first stage of labor

c. Evaluating the intensity by pressing the fingertips into the uterine fundus

Which statement, related to the reconditioning of the urinary system after childbirth, should the nurse understand? a. Kidney function returns to normal a few days after birth. b. Diastasis recti abdominis is a common condition that alters the voiding reflex. c. Fluid loss through perspiration and increased urinary output accounts for a weight loss of more than 2 kg during the puerperium. d. With adequate emptying of the bladder, bladder tone is usually restored 2 to 3 weeks after childbirth.

c. Fluid loss through perspiration and increased urinary output accounts for a weight loss of more than 2 kg during the puerperium.

The first hour after birth is sometimes referred to as what? a. Bonding period b. Third stage of labor c. Fourth stage of labor d. Early postpartum period

c. Fourth stage of labor

A woman's position is an important component of the labor progress. Which guidance is important for the nurse to provide to the laboring client? a. The supine position, which is commonly used in the United States, increases blood flow. b. The laboring client positioned on her hands and knees ("all fours" position) is hard on the woman's back. c. Frequent changes in position help relieve fatigue and increase the comfort of the laboring client. d. In a sitting or squatting position, abdominal muscles of the laboring client will have to work harder.

c. Frequent changes in position help relieve fatigue and increase the comfort of the laboring client.

Which documentation on a woman's chart on postpartum day 14 indicates a normal involution process? a. Moderate bright red lochial flow b. Breasts firm and tender c. Fundus below the symphysis and nonpalpable d. Episiotomy slightly red and puffy

c. Fundus below the symphysis and nonpalpable

What condition indicates concealed hemorrhage when the client experiences abruptio placentae? a. Decrease in abdominal pain b. Bradycardia c. Hard, boardlike abdomen d. Decrease in fundal height

c. Hard, boardlike abdomen

When a woman is diagnosed with postpartum depression (PPD) with psychotic features, what is the nurse's primary concern in planning the client's care? a. Displaying outbursts of anger b. Neglecting her hygiene c. Harming her infant d. Losing interest in her husband

c. Harming her infant

What behavior is the nurse most likely to assess in an adolescent with anorexia nervosa (AN)? a. Eats in secrecy b. Uses food as a coping mechanism c. Has a marked preoccupation with food d. Lacks awareness of how eating affects weight loss

c. Has a marked preoccupation with food

Nurses with an understanding of cultural differences regarding likely reactions to pain may be better able to help their clients. Which clients may initially appear very stoic but then become quite vocal as labor progresses until late in labor, when they become more vocal and request pain relief? a. Chinese b. Arab or Middle Eastern c. Hispanic d. African-American

c. Hispanic

When weighing the advantages and disadvantages of planning home care for perinatal services, what information should the nurse use in making the decision? a. Home care for perinatal services is more dangerous for vulnerable neonates at risk of acquiring an infection from the nurse. b. Home care for perinatal services is more cost-effective for the nurse than office visits. c. Home care for perinatal services allows the nurse to interact with and include family members in teaching. d. Home care for perinatal services is made possible by the ready supply of nurses with expertise in maternity care.

c. Home care for perinatal services allows the nurse to interact with and include family members in teaching.

The middle school nurse is planning a behavior modification program for overweight children. What is the most important goal for participants of the program? a. Learn how to cook low-fat meals. b. Improve relationships with peers. c. Identify and eliminate inappropriate eating habits. d. Achieve normal weight during the program.

c. Identify and eliminate inappropriate eating habits.

Dental care during pregnancy is an important component of good prenatal care. Which instruction regarding dental health should the nurse provide? a. Regular brushing and flossing may not be necessary during early pregnancy because it may stimulate the woman who is already nauseated to vomit. A cleaning is all that is necessary. b. Dental surgery is contraindicated during pregnancy and should be delayed until after delivery. c. If dental treatment is necessary, then the woman will be most comfortable with it in the second trimester. d. If a woman has dental anxiety, then dental care may interfere with the expectant mother's need to practice conscious relaxation and to prepare for labor.

c. If dental treatment is necessary, then the woman will be most comfortable with it in the second trimester.

The admission note indicates a patient diagnosed with major depressive disorder has anergia and anhedonia. For which measures should the nurse plan? (Select all that apply.) a. Channeling excessive energy b. Reducing guilty ruminations c. Instilling a sense of hopefulness d. Assisting with self-care activities e. Accommodating psychomotor retardation

c. Instilling a sense of hopefulness d. Assisting with self-care activities e. Accommodating psychomotor retardation

Which adaptation of the maternal-fetal exchange of oxygen occurs in response to uterine contraction? a. The maternal-fetal exchange of oxygen and waste products continues except when placental functions are reduced. b. This maternal-fetal exchange increases as the blood pressure decreases. c. It diminishes as the spiral arterioles are compressed. d. This exchange of oxygen and waste products is not significantly affected by contractions.

c. It diminishes as the spiral arterioles are compressed.

Two days ago a woman gave birth to a full-term infant. Last night she awakened several times to urinate and noted that her gown and bedding were wet from profuse diaphoresis. Which physiologic alteration is the cause for the diaphoresis and diuresis that this client is experiencing? a. Elevated temperature caused by postpartum infection b. Increased basal metabolic rate after giving birth c. Loss of increased blood volume associated with pregnancy d. Increased venous pressure in the lower extremities

c. Loss of increased blood volume associated with pregnancy

The nurse observes a sudden increase in variability on the electronic fetal monitoring (EFM) tracing. Which class of medications may cause this finding? a. Narcotics b. Barbiturates c. Methamphetamines d. Tranquilizers

c. Methamphetamines

The management of the pregnant client who has experienced a pregnancy loss depends on the type of miscarriage and the signs and symptoms. While planning care for a client who desires outpatient management after a first-trimester loss, what would the nurse expect the plan to include? a. Dilation and curettage (D&C) b. Dilation and evacuation (D&E) c. Misoprostol d. Ergot products

c. Misoprostol

A woman in labor has just received an epidural block. What is the most important nursing intervention at this time? a. Limit parenteral fluids. b. Monitor the fetus for possible tachycardia. c. Monitor the maternal blood pressure for possible hypotension. d. Monitor the maternal pulse for possible bradycardia.

c. Monitor the maternal blood pressure for possible hypotension.

A 14-year-old boy is of normal weight, and his parents are concerned about bilateral breast enlargement. The nurses discussion of this should be based on what? a. The presence of too much body fat b. Symptom that a hormonal imbalance is present c. Most likely part of normal pubertal development d. Indication that he is developing precocious puberty

c. Most likely part of normal pubertal development

A laboring woman has received meperidine intravenously (IV), 90 minutes before giving birth. Which medication should be available to reduce the postnatal effects of meperidine on the neonate? a. Fentanyl b. Promethazine c. Naloxone d. Nalbuphine

c. Naloxone

A labor and delivery nurse should be cognizant of which information regarding how the fetus moves through the birth canal? a. Fetal attitude describes the angle at which the fetus exits the uterus. b. Of the two primary fetal lies, the horizontal lie is that in which the long axis of the fetus is parallel to the long axis of the mother. c. Normal attitude of the fetus is called general flexion. d. Transverse lie is preferred for vaginal birth.

c. Normal attitude of the fetus is called general flexion.

A woman arrives at the clinic for a pregnancy test. Her last menstrual period (LMP) was February 14, 2019. What is the client's expected date of birth (EDB)? a. September 17, 2019 b. November 7, 2019 c. November 21, 2019 d. December 17, 2019

c. November 21, 2019

Which traditional family structure is decreasing in numbers and attributable to societal changes? a. Extended family b. Binuclear family c. Nuclear family d. Blended family

c. Nuclear family

Which condition is the least likely cause of amenorrhea in a 17-year-old client? a. Anatomic abnormalities b. Type 1 diabetes mellitus c. Obesity d. Pregnancy

c. Obesity

A client has a recurrent Bartholin cyst. What is the nurses priority action? a. Apply an ice pack to the area. b. Administer a prophylactic antibiotic. c. Obtain a fluid sample for laboratory analysis. d. Suggest moist heat such as a sitz bath.

c. Obtain a fluid sample for laboratory analysis.

Which statement regarding postpartum depression (PPD) is essential for the nurse to be aware of when attempting to formulate a nursing diagnosis? a. PPD symptoms are consistently severe. b. This syndrome affects only new mothers. c. PPD can easily go undetected. d. Only mental health professionals should teach new parents about this condition.

c. PPD can easily go undetected.

Descriptions of young people with anorexia nervosa (AN) often include which criteria? a. Impulsive b. Extroverted c. Perfectionist d. Low achieving

c. Perfectionist

The baseline fetal heart rate (FHR) is the average rate during a 10-minute segment. Changes in FHR are categorized as periodic or episodic. These patterns include both accelerations and decelerations. The labor nurse is evaluating the client's most recent 10-minute segment on the monitor strip and notes a late deceleration. Which is likely to have caused this change? (Select all that apply.) a. Spontaneous fetal movement b. Compression of the fetal head c. Placental abruption d. Cord around the baby's neck e. Maternal supine hypotension

c. Placental abruption e. Maternal supine hypotension

During a telephone follow-up conversation with a woman who is 4 days postpartum, the woman tells the nurse, "I don't know what's wrong. I love my son, but I feel so let down. I seem to cry for no reason!" Which condition might this new mother be experiencing? a. Letting-go b. Postpartum depression (PPD) c. Postpartum blues d. Attachment difficulty

c. Postpartum blues

While providing care to the maternity client, the nurse should be aware that one of these anxiety disorders is likely to be triggered by the process of labor and birth. Which disorder fits this criterion? a. Phobias b. Panic disorder c. Posttraumatic stress disorder (PTSD) d. Obsessive-compulsive disorder (OCD)

c. Posttraumatic stress disorder (PTSD)

The emergency department nursing assessment of a pregnant female at 35 weeks gestation reveals back pain, blood pressure 150/92, and leaking of clear fluid from the vagina. Which complication of pregnancy does the nurse suspect? a. Ectopic pregnancy b. Spontaneous abortion c. Premature rupture of membranes d. Supine hypotension

c. Premature rupture of membranes

Which hormone remains elevated in the immediate postpartum period of the breastfeeding woman? a. Estrogen b. Progesterone c. Prolactin d. Human placental lactogen

c. Prolactin

In planning for home care of a woman with preterm labor, which concern should the nurse need to address? a. Nursing assessments are different from those performed in the hospital setting. b. Restricted activity and medications are necessary to prevent a recurrence of preterm labor. c. Prolonged bed rest may cause negative physiologic effects. d. Home health care providers are necessary.

c. Prolonged bed rest may cause negative physiologic effects.

In which clinical situation would the nurse most likely anticipate a fetal bradycardia? a. Intraamniotic infection b. Fetal anemia c. Prolonged umbilical cord compression d. Tocolytic treatment using terbutaline

c. Prolonged umbilical cord compression

The nurses role in facilitating successful childrearing in unmarried teenage mothers includes what? a. Facilitating marriage between the mother and father of the baby b. Teaching the adolescent the long-term needs of the growing child c. Providing information and feedback about positive parenting skills d. Encouraging the infants grandmother to take responsibility for care

c. Providing information and feedback about positive parenting skills

When assessing the fetus using Leopold's maneuvers, the nurse feels a round, firm, and movable fetal part in the fundal portion of the uterus and a long, smooth surface in the mother's right side close to midline. What is the position of the fetus? a. ROA b. LSP c. RSA d. LOA

c. RSA

What should the nurse's next action be if the client's white blood cell (WBC) count is 25,000/mm3 on her second postpartum day? a. Immediately inform the health care provider. b. Have the laboratory draw blood for reanalysis. c. Recognize that this count is an acceptable range at this point postpartum. d. Immediately begin antibiotic therapy.

c. Recognize that this count is an acceptable range at this point postpartum.

What is the primary rationale for the thorough drying of the infant immediately after birth? a. Stimulates crying and lung expansion b. Removes maternal blood from the skin surface c. Reduces heat loss from evaporation d. Increases blood supply to the hands and feet

c. Reduces heat loss from evaporation

What statement is true about smoking in college students? a. The rate of smoking cigarettes is declining. b. Smokeless tobacco use is rising dramatically. c. Regular cigar use is becoming more common. d. Students in the health professions do not smoke.

c. Regular cigar use is becoming more common.

A nurse is developing a plan of care for a patient admitted with a diagnosis of bipolar disorder, manic phase. Which nursing diagnoses address priority needs for the patient? (Select all that apply.) a. Risk for caregiver strain b. Impaired verbal communication c. Risk for injury d. Imbalanced nutrition, less than body requirements e. Ineffective coping f. Sleep deprivation

c. Risk for injury d. Imbalanced nutrition, less than body requirements f. Sleep deprivation

A woman is having her first child. She has been in labor for 15 hours. A vaginal examination performed 2 hours earlier revealed the cervix to be dilated to 5 cm and 100% effaced, and the presenting part of the fetus was at station 0; however, another vaginal examination performed 5 minutes ago indicated no changes. What abnormal labor pattern is associated with this description? a. Prolonged latent phase b. Protracted active phase c. Secondary arrest d. Protracted descent

c. Secondary arrest

A pregnant woman who abuses cocaine admits to exchanging sex to finance her drug habit. This behavior places the client at the greatest risk for what? a. Depression of the CNS b. Hypotension and vasodilation c. Sexually transmitted infections (STIs) d. Postmature birth

c. Sexually transmitted infections (STIs)

A woman has requested an epidural for her pain. She is 5 cm dilated and 100% effaced. The baby is in a vertex position and is engaged. The nurse increases the woman's IV fluid for a preprocedural bolus. The nurse reviews her laboratory values and notes that the woman's hemoglobin is 12 g/dl, hematocrit is 38%, platelets are 67,000, and white blood cells (WBCs) are 12,000/mm3. Which factor would contraindicate an epidural for this woman? a. She is too far dilated. b. She is anemic. c. She has thrombocytopenia. d. She is septic.

c. She has thrombocytopenia.

A primigravida at 40 weeks of gestation is having uterine contractions every to 2 minutes and states that they are very painful. Her cervix is dilated 2 cm and has not changed in 3 hours. The woman is crying and wants an epidural. What is the likely status of this woman's labor? a. She is exhibiting hypotonic uterine dysfunction. b. She is experiencing a normal latent stage. c. She is exhibiting hypertonic uterine dysfunction. d. She is experiencing precipitous labor.

c. She is exhibiting hypertonic uterine dysfunction.

Reports have linked third trimester use of selective serotonin uptake inhibitors (SSRIs) with a constellation of neonatal signs. The nurse is about to perform an assessment on the infant of a mother with a history of a mood disorder. Which signs and symptoms in the neonate may be the result of maternal SSRI use? (Select all that apply.) a. Hypotonia b. Hyperglycemia c. Shivering d. Fever e. Irritability

c. Shivering d. Fever e. Irritability

Which principle should guide the nurse in determining the extent of silence to use during patient interview sessions? a. A nurse is responsible for breaking silences. b. Patients withdraw if silences are prolonged. c. Silence can provide meaningful moments for reflection. d. Silence helps patients know that what they said was understood.

c. Silence can provide meaningful moments for reflection.

Major depressive disorder resulted after a patient's employment was terminated. The patient now says to the nurse, "I'm not worth the time you spend with me. I am the most useless person in the world." Which nursing diagnosis applies? a. Powerlessness b. Defensive coping c. Situational low self-esteem d. Disturbed personal identity

c. Situational low self-esteem

A woman who is 30 weeks of gestation arrives at the hospital with bleeding. Which differential diagnosis would not be applicable for this client? a. Placenta previa b. Abruptio placentae c. Spontaneous abortion d. Cord insertion

c. Spontaneous abortion

The most effective and least expensive treatment of puerperal infection is prevention. What is the most important strategy for the nurse to adopt? a. Large doses of vitamin C during pregnancy b. Prophylactic antibiotics c. Strict aseptic technique by all health care personnel d. Limited protein and fat intake

c. Strict aseptic technique by all health care personnel

A patient diagnosed with major depressive disorder received six ECT sessions and aggressive doses of antidepressant medication. The patient owns a small business and was counseled not to make major decisions for a month. Select the correct rationale for this counseling. a. Antidepressant medications alter catecholamine levels, which impairs decision-making abilities. b. Antidepressant medications may cause confusion related to limitation of tyramine in the diet. c. Temporary memory impairments and confusion may occur with ECT. d. The patient needs time to readjust to a pressured work schedule.

c. Temporary memory impairments and confusion may occur with ECT.

The nurse has determined that an adolescents body mass index (BMI) is in the 90th percentile. What information should the nurse convey to the adolescent? a. The adolescent is overweight. b. The adolescent has maintained weight within the normal range. c. The adolescent is at risk for becoming overweight. d. Nutritional supplementation should occur at least three times per week

c. The adolescent is at risk for becoming overweight.

What is the primary purpose for the use of tocolytic therapy to suppress uterine activity? a. Drugs can be efficaciously administered up to the designated beginning of term at 37 weeks gestation. b. Tocolytic therapy has no important maternal (as opposed to fetal) contraindications. c. The most important function of tocolytic therapy is to provide the opportunity to administer antenatal glucocorticoids. d. If the client develops pulmonary edema while receiving tocolytic therapy, then intravenous (IV) fluids should be given.

c. The most important function of tocolytic therapy is to provide the opportunity to administer antenatal glucocorticoids.

During the initial visit with a client who is beginning prenatal care, which action should be the highest priority for the nurse? a. The first interview is a relaxed, get-acquainted affair during which the nurse gathers some general impressions of his or her new client. b. If the nurse observed handicapping conditions, he or she should be sensitive and not inquire about them because the client will do that in her own time. c. The nurse should be alert to the appearance of potential parenting problems, such as depression or lack of family support. d. Because of legal complications, the nurse should not ask about illegal drug use; that is left to the physician.

c. The nurse should be alert to the appearance of potential parenting problems, such as depression or lack of family support.

Documentation in a patient's chart shows, "Throughout a 5-minute interaction, patient fidgeted and tapped left foot, periodically covered face with hands, and looked under chair while stating, 'I enjoy spending time with you.'" Which analysis is most accurate? a. The patient is giving positive feedback about the nurse's communication techniques. b. The nurse is viewing the patient's behavior through a cultural filter. c. The patient's verbal and nonverbal messages are incongruent. d. The patient is demonstrating psychotic behaviors.

c. The patient's verbal and nonverbal messages are incongruent.

What is a distinct advantage of external electronic fetal monitoring (EFM)? a. The ultrasound transducer can accurately measure short-term variability and beat-to-beat changes in the fetal heart rate (FHR). b. The toco transducer can measure and record the frequency, regularity, intensity, and approximate duration of uterine contractions. c. The toco transducer is especially valuable for measuring uterine activity during the first stage of labor. d. Once correctly applied by the nurse, the transducer need not be repositioned even when the woman changes positions.

c. The toco transducer is especially valuable for measuring uterine activity during the first stage of labor.

A recently delivered mother and her baby are at the clinic for a 6-week postpartum checkup. Which response by the client alerts the nurse that psychosocial outcomes have not been met? a. The woman excessively discusses her labor and birth experience. b. The woman feels that her baby is more attractive and cleverer than any others. c. The woman refers to the baby as "sweetie" since she has not given the baby a name yet. d. The woman has a partner or family members who react very positively about the baby.

c. The woman refers to the baby as "sweetie" since she has not given the baby a name yet.

A woman arrives at the emergency department with reports of bleeding and cramping. The initial nursing history is significant for a last menstrual period 6 weeks ago. On sterile speculum examination, the primary care provider finds that the cervix is closed. The anticipated plan of care for this woman would be based on a probable diagnosis of which type of spontaneous abortion? a. Incomplete b. Inevitable c. Threatened d. Septic

c. Threatened

A woman who has recently given birth reports pain and tenderness in her leg. On physical examination, the nurse notices warmth and redness over an enlarged, hardened area. Which condition should the nurse suspect, and how will it be confirmed? a. Disseminated intravascular coagulation (DIC); asking for laboratory tests b. von Willebrand disease (vWD); noting whether bleeding times have been extended c. Thrombophlebitis; using real-time and color Doppler ultrasound d. Idiopathic or immune thrombocytopenic purpura (ITP); drawing blood for laboratory analysis

c. Thrombophlebitis; using real-time and color Doppler ultrasound

Prostaglandin gel has been ordered for a pregnant woman at 43 weeks of gestation. What is the primary purpose of prostaglandin administration? a. To enhance uteroplacental perfusion in an aging placenta b. To increase amniotic fluid volume c. To ripen the cervix in preparation for labor induction d. To stimulate the amniotic membranes to rupture

c. To ripen the cervix in preparation for labor induction

Which substance used during pregnancy causes vasoconstriction and decreased placental perfusion, resulting in maternal and neonatal complications? a. Alcohol b. Caffeine c. Tobacco d. Chocolate

c. Tobacco

The nurse is preparing to administer methotrexate to the client. This hazardous drug is most often used for which obstetric complication? a. Complete hydatidiform mole b. Missed abortion c. Unruptured ectopic pregnancy d. Abruptio placentae

c. Unruptured ectopic pregnancy

A 25-year-old gravida 3, para 2 client gave birth to a 9-pound, 7-ounce boy, 4 hours ago after augmentation of labor with oxytocin. She presses her call light, and asks for her nurse right away, stating "I'm bleeding a lot." What is the most likely cause of postpartum hemorrhaging in this client? a. Retained placental fragments b. Unrepaired vaginal lacerations c. Uterine atony d. Puerperal infection

c. Uterine atony

The nurse providing care for a high-risk laboring woman is alert for late fetal heart rate (FHR) decelerations. Which clinical finding might be the cause for these late decelerations? a. Altered cerebral blood flow b. Umbilical cord compression c. Uteroplacental insufficiency d. Meconium fluid

c. Uteroplacental insufficiency

The nurse who provides care to clients in labor must have a thorough understanding of the physiologic processes of maternal hypotension. Which outcome might occur if the interventions for maternal hypotension are inadequate? a. Early fetal heart rate (FHR) decelerations b. Fetal arrhythmias c. Uteroplacental insufficiency d. Spontaneous rupture of membranes

c. Uteroplacental insufficiency

Which nursing assessment indicates that a woman who is in second-stage labor is almost ready to give birth? a. Fetal head is felt at 0 station during the vaginal examination. b. Bloody mucous discharge increases. c. Vulva bulges and encircles the fetal head. d. Membranes rupture during a contraction.

c. Vulva bulges and encircles the fetal head.

Which nursing assessment indicates that a woman who is in second-stage labor is almost ready to give birth? a. Fetal head is felt at 0 station during vaginal examination. b. Bloody mucous discharge increases. c. Vulva bulges and encircles the fetal head. d. Membranes rupture during a contraction.

c. Vulva bulges and encircles the fetal head.

Which clinical finding indicates that the client has reached the second stage of labor? a. Amniotic membranes rupture (ROM). b. Cervix cannot be felt during a vaginal examination. c. Woman experiences a strong urge to bear down. d. Presenting part of the fetus is below the ischial spines.

c. Woman experiences a strong urge to bear down.

What should the nurse be cognizant of concerning the client's reordering of personal relationships during pregnancy? a. Because of the special motherhood bond, a woman's relationship with her mother is even more important than with her partner. b. Nurses need not get involved in any sexual issues the couple has during pregnancy, particularly if they have trouble communicating them to each other. c. Women usually express two major relationship needs during pregnancy: feeling loved and valued and having the child accepted by the partner. d. The woman's sexual desire is likely to be highest in the first trimester because of the excitement and because intercourse is physically easier.

c. Women usually express two major relationship needs during pregnancy: feeling loved and valued and having the child accepted by the partner.

When counseling patients diagnosed with major depressive disorder, an advanced practice nurse will address the negative thought patterns by using a. psychoanalytic therapy. b. desensitization therapy. c. cognitive-behavioral therapy. d. alternative and complementary therapies.

c. cognitive-behavioral therapy.

A patient diagnosed with major depressive disorder began taking escitalopram 5 days ago. The patient now says, "This medicine isn't working." The nurse's best intervention would be to a. discuss with the health care provider the need to increase the dose. b. reassure the patient that the medication will be effective soon. c. explain the time lag before antidepressants relieve symptoms. d. critically assess the patient for symptoms of improvement.

c. explain the time lag before antidepressants relieve symptoms.

A patient diagnosed with depressive disorder begins selective serotonin reuptake inhibitor (SSRI) antidepressant therapy. The nurse should provide information to the patient and family about a. restricting sodium intake to 1 gram daily. b. minimizing exposure to bright sunlight. c. reporting increased suicidal thoughts. d. maintaining a tyramine-free diet.

c. reporting increased suicidal thoughts.

A patient diagnosed with major depressive disorder began taking a tricyclic antidepressant 1 week ago. Today the patient says, "I don't think I can keep taking these pills. They make me so dizzy, especially when I stand up." The nurse will a. limit the patient's activities to those that can be performed in a sitting position. b. withhold the drug, force oral fluids, and notify the health care provider. c. teach the patient strategies to manage postural hypotension. d. update the patient's mental status examination.

c. teach the patient strategies to manage postural hypotension.

A woman who is 14 weeks pregnant tells the nurse that she always had a glass of wine with dinner before she became pregnant. She has abstained during her first trimester and would like to know if it is safe for her to have a drink with dinner now. Which guidance should the nurse provide? a. "Since you're in your second trimester, there's no problem with having one drink with dinner." b. "One drink every night is too much. One drink three times a week should be fine." c. "Since you're in your second trimester, you can drink as much as you like." d. "Because no one knows how much or how little alcohol it takes to cause fetal problems, the best course is to abstain throughout your pregnancy."

d. "Because no one knows how much or how little alcohol it takes to cause fetal problems, the best course is to abstain throughout your pregnancy."

A patient says to the nurse, "I dreamed I was stoned. When I woke up, I felt emotionally drained, as though I hadn't rested well." Which response should the nurse use to clarify the patient's comment? a. "It sounds as though you were uncomfortable with the content of your dream." b. "I understand what you're saying. Bad dreams leave me feeling tired, too." c. "So you feel as though you did not get enough quality sleep last night?" d. "Can you give me an example of what you mean by 'stoned'?"

d. "Can you give me an example of what you mean by 'stoned'?"

A patient states, "I had a bad nightmare. When I woke up, I felt emotionally drained, as though I hadn't rested well." Which response by the nurse would be an example of interpersonal therapeutic communication? a. "It sounds as though you were uncomfortable with the content of your dream." b. "I understand what you're saying. Bad dreams leave me feeling tired, too." c. "So, all in all, you feel as though you had a rather poor night's sleep?" d. "Can you give me an example of what you mean by a 'bad nightmare'?"

d. "Can you give me an example of what you mean by a 'bad nightmare'?"

A woman is 3 months pregnant. At her prenatal visit she tells the nurse that she does not know what is happening; one minute she is happy that she is pregnant and the next minute she cries for no reason. Which response by the nurse is most appropriate? a. "Don't worry about it; you'll feel better in a month or so." b. "Have you talked to your husband about how you feel?" c. "Perhaps you really don't want to be pregnant." d. "Hormone changes during pregnancy commonly result in mood swings."

d. "Hormone changes during pregnancy commonly result in mood swings."

What represents a typical progression through the phases of a woman's establishing a relationship with the fetus? a. Accepts the fetus as distinct from herself—accepts the biologic fact of pregnancy—has feelings of caring and responsibility. b. Fantasizes about the child's gender and personality—views the child as part of herself—becomes introspective. c. Views the child as part of herself—has feelings of well-being—accepts the biologic fact of the pregnancy. d. "I am pregnant"—"I am going to have a baby"—"I am going to be a mother."

d. "I am pregnant"—"I am going to have a baby"—"I am going to be a mother."

A nurse interacts with a newly hospitalized patient. Select the nurse's comment that applies the communication technique of "offering self." a. "I've also had traumatic life experiences. Maybe it would help if I told you about them." b. "Why do you think you had so much difficulty adjusting to this change in your life?" c. "I hope you will feel better after getting accustomed to how this unit operates." d. "I'd like to sit with you for a while to help you get comfortable talking to me."

d. "I'd like to sit with you for a while to help you get comfortable talking to me."

The nurse is admitting a patient to the medical/surgical unit. Which communication technique would be considered appropriate for this interaction? a. "I've also had traumatic life experiences. Maybe it would help if I told you about them." b. "Why do you think you had so much difficulty adjusting to this change in your life?" c. "You will feel better after getting accustomed to how this unit operates." d. "I'd like to sit with you for a while to help you get comfortable talking to me."

d. "I'd like to sit with you for a while to help you get comfortable talking to me."

A patient became severely depressed when the last of the family's six children moved out of the home 4 months ago. The patient repeatedly says, "No one cares about me. I'm not worth anything." Which response by the nurse would be the most helpful? a. "Things will look brighter soon. Everyone feels down once in a while." b. "Our staff members care about you and want to try to help you get better." c. "It is difficult for others to care about you when you repeatedly say the same negative things." d. "I'll sit with you for 10 minutes now and 10 minutes after lunch to help you feel that I care about you."

d. "I'll sit with you for 10 minutes now and 10 minutes after lunch to help you feel that I care about you."

A 25-year-old multiparous woman gave birth to an infant boy 1 day ago. Today her husband brings a large container of brown seaweed soup to the hospital. When the nurse enters the room, the husband asks for help with warming the soup so that his wife can eat it. What is the nurse's most appropriate response? a. "Didn't you like your lunch?" b. "Does your health care provider know that you are planning to eat that?" c. "What is that anyway?" d. "I'll warm the soup in the microwave for you."

d. "I'll warm the soup in the microwave for you."

A patient newly diagnosed with depression states, "I have had other people in my family say that they have depression. Is this an inherited problem?" What is the nurse's best response? a. "There are a lot of mood disorders that are caused by many different causes. Inheriting these disorders is not likely." b. "Current research is focusing on fluid and electrolyte disorders as a cause for mood disorders." c. "All of your family members raised in the same area have probably learned to respond to problems in the same way." d. "Members of the same family may have the same biological predisposition to experiencing mood disorders."

d. "Members of the same family may have the same biological predisposition to experiencing mood disorders."

A pregnant couple has formulated a birth plan and is reviewing it with the nurse at an expectant parent's class. Which aspect of their birth plan should be considered potentially unrealistic and require further discussion with the nurse? a. "My partner and I have agreed that my sister will be my coach because he becomes anxious with regard to medical procedures and blood. He will be nearby and check on me every so often to make sure everything is okay." b. "We plan to use the techniques taught in the Lamaze classes to reduce the pain experienced during labor." c. "We want the labor and birth to take place in a birthing room. My partner will come in the minute the baby is born." d. "Regardless of the circumstances, we do not want the fetal monitor used during labor because it will interfere with movement and doing effleurage."

d. "Regardless of the circumstances, we do not want the fetal monitor used during labor because it will interfere with movement and doing effleurage."

school age child tells the school nurse, "Other kids call me mean names and will not sit with me at lunch. Nobody likes me." Select the nurse's most therapeutic response. a. "Just ignore them and they will leave you alone." b. "You should make friends with other children." c. "Call them names if they do that to you." d. "Tell me more about how you feel."

d. "Tell me more about how you feel."

During an interview, a patient attempts to shift the focus from self to the nurse by asking personal questions. The nurse should respond by saying: a. "Why do you keep asking about me?" b. "Nurses direct the interviews with patients." c. "Do not ask questions about my personal life." d. "The time we spend together is to discuss your concerns."

d. "The time we spend together is to discuss your concerns."

A client is in early labor, and her nurse is discussing the pain relief options she is considering. The client states that she wants an epidural "no matter what!" What is the nurse's best response? a. "I'll make sure you get your epidural." b. "You may only have an epidural if your physician allows it." c. "You may only have an epidural if you are going to deliver vaginally." d. "The type of analgesia or anesthesia used is determined, in part, by the stage of your labor and the method of birth."

d. "The type of analgesia or anesthesia used is determined, in part, by the stage of your labor and the method of birth."

A patient who is taking prescribed lithium carbonate is exhibiting signs of diarrhea, blurred vision, frequent urination, and an unsteady gait. Which serum lithium level would the nurse expect for this patient? a. 0 to 0.5 mEq/L b. 0.6 to 0.9 mEq/L c. 1.0 to 1.4 mEq/L d. 1.5 or higher mEq/L

d. 1.5 or higher mEq/L

A new mother asks the nurse when the "soft spot" on her son's head will go away. What is the nurse's best response, based upon her understanding of when the anterior frontal closes? a. 2 months b. 8 months c. 12 months d. 18 months

d. 18 months

Parents who have not already done so need to make time for newborn follow-up of the discharge. According to the American Academy of Pediatrics (AAP), when should a breastfeeding infant first need to be seen for a follow-up examination? a. 2 weeks of age b. 7 to 10 days after childbirth c. 4 to 5 days after hospital discharge d. 48 to 72 hours after hospital discharge

d. 48 to 72 hours after hospital discharge

The nurse is presenting an educational program to a group of parents about differences between anorexia nervosa (AN) and bulimia nervosa (BN) at a community outreach program. What statement by a parent would indicate a need for additional teaching? a. A child with AN will turn away from food to cope, but a child with BN turns to food to cope. b. A child with AN maintains rigid control and is introverted, but a child with BN is an extrovert and frequently loses control. c. A child with AN denies the illness, but a child with BN recognizes the illness. d. A child with AN is usually sexually active and seeks intimacy, but a child with BN avoids intimacy and is usually not sexually active.

d. A child with AN is usually sexually active and seeks intimacy, but a child with BN avoids intimacy and is usually not sexually active.

A 25-year-old gravida 1 para 1 who had an emergency cesarean birth 3 days ago is scheduled for discharge. As the nurse prepares her for discharge, she begins to cry. The nurse's next action should be what? a. Assess her for pain. b. Point out how lucky she is to have a healthy baby. c. Explain that she is experiencing postpartum blues. d. Allow her time to express her feelings.

d. Allow her time to express her feelings.

Which definition of an acceleration in the fetal heart rate (FHR) is accurate? a. FHR accelerations are indications of fetal well-being when they are periodic. b. FHR accelerations are greater and longer in preterm gestations. c. FHR accelerations are usually observed with breech presentations when they are episodic. d. An acceleration in the FHR presents a visually apparent and abrupt peak.

d. An acceleration in the FHR presents a visually apparent and abrupt peak.

While providing routine mother-baby care, which activities should the nurse encourage to facilitate the parent-infant attachment? a. The baby is able to return to the nursery at night so that the new mother can sleep. b. Routine times for care are established to reassure the parents. c. The father should be encouraged to go home at night to prepare for discharge of the mother and baby. d. An environment that fosters as much privacy as possible should be created.

d. An environment that fosters as much privacy as possible should be created.

Which characteristic of a uterine contraction is not routinely documented? a. Frequency: how often contractions occur b. Intensity: strength of the contraction at its peak c. Resting tone: tension in the uterine muscle d. Appearance: shape and height

d. Appearance: shape and height

When a nulliparous woman telephones the hospital to report that she is in labor, what intervention should the nurse prioritize? a. Instructing the woman to stay home until her membranes rupture. b. Emphasizing that food and fluid intake should stop. c. Arranging for the woman to come to the hospital for labor evaluation. d. Asking the woman to describe why she believes she is in labor.

d. Asking the woman to describe why she believes she is in labor.

The obstetric nurse is preparing the client for an emergency cesarean birth, with no time to administer spinal anesthesia, general anesthesia will be used. What is the greatest risk of administering general anesthesia to the client.? a. Respiratory depression b. Uterine relaxation c. Inadequate muscle relaxation d. Aspiration of stomach contents

d. Aspiration of stomach contents

A multiparous woman has been in labor for 8 hours. Her membranes have just ruptured. What is the nurse's highest priority in this situation? a. Prepare the woman for imminent birth. b. Notify the woman's primary health care provider. c. Document the characteristics of the fluid. d. Assess the fetal heart rate (FHR) and pattern.

d. Assess the fetal heart rate (FHR) and pattern.

The obstetric provider has informed the nurse that she will be performing an amniotomy on the client to induce labor. What is the nurse's highest priority intervention after the amniotomy is performed? a. Applying clean linens under the woman b. Taking the client's vital signs c. Performing a vaginal examination d. Assessing the fetal heart rate (FHR)

d. Assessing the fetal heart rate (FHR)

A woman who is pregnant for the first time is dilated 3 cm and having contractions every 5 minutes. She is groaning and perspiring excessively; she states that she did not attend childbirth classes. What is the optimal intervention for the nurse to provide initially? a. Notify the woman's health care provider. b. Administer the prescribed narcotic analgesic. c. Assure her that her labor will be over soon. d. Assist her with simple breathing and relaxation instructions.

d. Assist her with simple breathing and relaxation instructions.

After birth, a crying infant may be soothed by being held in a position in which the newborn can hear the mother's heartbeat. This phenomenon is known as what? a. Entrainment b. Reciprocity c. Synchrony d. Biorhythmicity

d. Biorhythmicity

As a powerful central nervous system (CNS) stimulant, which of these substances can lead to miscarriage, preterm labor, placental separation (abruption), and stillbirth? a. Heroin b. Alcohol c. Phencyclidine (1-phenylcyclohexylpiperidine; PCP) d. Cocaine

d. Cocaine

An obstetric multipara with triplets is placed on bed rest at 24 weeks' gestation. Her perinatologist is managing intrauterine growth restriction with serial ultrasounds. This prescribed treatment is an example of which type of care? a. Antenatal diagnostics b. Primary prevention c. Secondary prevention d. Collaborative intervention

d. Collaborative intervention

A client is concerned that her breasts are engorged and uncomfortable. What is the nurse's explanation for this physiologic change? a. Overproduction of colostrum b. Accumulation of milk in the lactiferous ducts and glands c. Hyperplasia of mammary tissue d. Congestion of veins and lymphatic vessels

d. Congestion of veins and lymphatic vessels

The client is emotionally upset about the recent diagnosis of stage IV endometrial cancer. Which action by the nurse is best? a. Let the client alone for a long period of reflection time. b. Ask friends and relatives to limit their visits. c. Tell the client that an emotional response is unacceptable. d. Create an atmosphere of acceptance and discussion.

d. Create an atmosphere of acceptance and discussion.

The nurse is performing an assessment on a client who thinks she may be experiencing preterm labor. Which information is the most important for the nurse to understand and share with the client? a. Because all women must be considered at risk for preterm labor and prediction is so variable, teaching pregnant women the symptoms of preterm labor probably causes more harm through false alarms. b. Braxton Hicks contractions often signal the onset of preterm labor. c. Because preterm labor is likely to be the start of an extended labor, a woman with symptoms can wait several hours before contacting the primary caregiver. d. Diagnosis of preterm labor is based on gestational age, uterine activity, and progressive cervical change.

d. Diagnosis of preterm labor is based on gestational age, uterine activity, and progressive cervical change.

If nonsurgical treatment for late postpartum hemorrhage (PPH) is ineffective, which surgical procedure would be appropriate to correct the cause of this condition? a. Hysterectomy b. Laparoscopy c. Laparotomy d. Dilation and curettage (D&C)

d. Dilation and curettage (D&C)

A primigravida at 39 weeks of gestation is observed for 2 hours in the intrapartum unit. The fetal heart rate (FHR) has been normal. Contractions are 5 to 9 minutes apart, 20 to 30 seconds in duration, and of mild intensity. Cervical dilation is 1 to 2 cm and uneffaced (unchanged from admission). Membranes are intact. What disposition would the nurse anticipate? a. Admitted and prepared for a cesarean birth b. Admitted for extended observation c. Discharged home with a sedative d. Discharged home to await the onset of true labor

d. Discharged home to await the onset of true labor

The nurse performs a vaginal examination to assess a client's labor progress. Which action should the nurse take next? a. Perform an examination at least once every hour during the active phase of labor. b. Perform the examination with the woman in the supine position. c. Wear two clean gloves for each examination. d. Discuss the findings with the woman and her partner.

d. Discuss the findings with the woman and her partner.

The nurse is caring for a postoperative client following an anterior colporrhaphy. What action can be delegated to the unlicensed assistive personnel (UAP)? a. Reviewing the hematocrit and hemoglobin results b. Teaching the client to avoid lifting her 4-year-old grandson c. Assessing the level of pain and any drainage d. Drawing a shallow hot bath for comfort measures

d. Drawing a shallow hot bath for comfort measures

The Valsalva maneuver can be described as the process of making a forceful bearing-down attempt while holding one's breath with a closed glottis and a tightening of the abdominal muscles. When is it appropriate to instruct the client to use this maneuver? a. During the second stage to enhance the movement of the fetus b. During the third stage to help expel the placenta c. During the fourth stage to expel blood clots d. During no stage of labor

d. During no stage of labor

Maternity nurses often must answer questions about the many, sometimes unusual, ways people have tried to make the birthing experience more comfortable. Which information regarding nonpharmacologic pain relief is accurate? a. Music supplied by the support person must be discouraged because it could disturb others or upset the hospital routine. b. Women in labor can benefit from sitting in a bathtub, but they must limit immersion to no longer than 15 minutes at a time. c. Effleurage is permissible, but counterpressure is almost always counterproductive. d. Electrodes attached to either side of the spine to provide high-intensity electrical impulses facilitate the release of endorphins.

d. Electrodes attached to either side of the spine to provide high-intensity electrical impulses facilitate the release of endorphins.

A client has just returned from a total abdominal hysterectomy and needs postoperative nursing care. What action can the nurse delegate to the unlicensed assistive personnel (UAP)? a. Assess heart, lung, and bowel sounds. b. Check the hemoglobin and hematocrit levels. c. Evaluate the dressing for drainage. d. Empty the urine from the urinary catheter bag.

d. Empty the urine from the urinary catheter bag.

Which statement regarding the care of a client in labor is correct and important to the nurse in the formulation of the plan of care? a. The woman's blood pressure will increase during contractions and fall back to prelabor normal levels between contractions. b. The use of the Valsalva maneuver is encouraged during the second stage of labor to relieve fetal hypoxia. c. Having the woman point her toes will reduce leg cramps. d. Endogenous endorphins released during labor will raise the woman's pain threshold and produce sedation.

d. Endogenous endorphins released during labor will raise the woman's pain threshold and produce sedation.

During the physical examination of an adolescent with significant weight loss, what finding may indicate an eating disorder? a. Diarrhea b. Amenorrhea c. Appetite suppression d. Erosion of tooth enamel

d. Erosion of tooth enamel

What physiologic change occurs as the result of increasing the infusion rate of nonadditive IV fluids? a. Maintaining normal maternal temperature b. Preventing normal maternal hypoglycemia c. Increasing the oxygen-carrying capacity of the maternal blood d. Expanding maternal blood volume

d. Expanding maternal blood volume

Certain changes stimulate chemoreceptors in the aorta and carotid bodies to prepare the fetus for initiating respirations immediately after birth. Which change in fetal physiologic activity is not part of this process? a. Fetal lung fluid is cleared from the air passages during labor and vaginal birth. b. Fetal partial pressure of oxygen (PO2) decreases. c. Fetal partial pressure of carbon dioxide in arterial blood (PaCO2) increases. d. Fetal respiratory movements increase during labor.

d. Fetal respiratory movements increase during labor.

Which collection of risk factors will most likely result in damaging lacerations, including episiotomies? a. Dark-skinned woman who has had more than one pregnancy, who is going through prolonged second-stage labor, and who is attended by a midwife b. Reddish-haired mother of two who is going through a breech birth c. Dark-skinned first-time mother who is going through a long labor d. First-time mother with reddish hair whose rapid labor was overseen by an obstetrician

d. First-time mother with reddish hair whose rapid labor was overseen by an obstetrician

While assessing the vital signs of a pregnant woman in her third trimester, the client reports feeling faint, dizzy, and agitated. Which nursing intervention is appropriate? a. Have the patient stand up, and then retake her BP. b. Have the patient sit down, and then hold her arm in a dependent position. c. Have the patient lie supine for 5 minutes, and then recheck her BP on both arms. d. Have the patient turn to her left side, and then recheck her BP in 5 minutes.

d. Have the patient turn to her left side, and then recheck her BP in 5 minutes.

Which condition, not uncommon in pregnancy, is likely to require careful medical assessment during the puerperium? a. Varicosities of the legs b. Carpal tunnel syndrome c. Periodic numbness and tingling of the fingers d. Headaches

d. Headaches

woman in the active phase of the first stage of labor is using a shallow pattern of breathing, which is approximately twice the normal adult breathing rate. She starts to report feeling lightheaded and dizzy and states that her fingers are tingling. Which intervention should the nurse immediately initiate? a. Contact the woman's health care provider. b. Tell the woman to slow her pace of her breathing. c. Administer oxygen via a mask or nasal cannula. d. Help her breathe into a paper bag.

d. Help her breathe into a paper bag.

A 32-year-old primigravida is admitted with a diagnosis of ectopic pregnancy. Which information assists the nurse in developing the plan of care? a. Bed rest and analgesics are the recommended treatment. b. She will be unable to conceive in the future. c. A D&C will be performed to remove the products of conception. d. Hemorrhage is the primary concern.

d. Hemorrhage is the primary concern.

What is a limitation of a home postpartum visit? a. Distractions limit the nurse's ability to teach. b. Identified problems cannot be resolved in the home setting. c. Necessary items for infant care are not available. d. Home visits to different families may require the nurse to travel a great distance.

d. Home visits to different families may require the nurse to travel a great distance.

Which statement regarding the condition referred to as a miscarriage is most accurate? a. A miscarriage is a natural pregnancy loss before labor begins. b. It occurs in fewer than 5% of all clinically recognized pregnancies. c. Careless maternal behavior, such as poor nutrition or excessive exercise, can be a factor in causing a miscarriage. d. If a miscarriage occurs before the 12th week of pregnancy, then it may be observed only as moderate discomfort and blood loss.

d. If a miscarriage occurs before the 12th week of pregnancy, then it may be observed only as moderate discomfort and blood loss.

The nurse is evaluating the electronic feta monitoring (EFM) tracing of the client who is in active labor. Suddenly, the fetal heart rate (FHR) drops from its baseline of 125 down to 80 beats per minute. The mother is repositioned, and the nurse provides oxygen, increased IV fluids, and performs a vaginal examination. The cervix has not changed. Five minutes have passed, and the FHR remains in the 80s. What additional nursing measures should the nurse take next? a. Call for help. b. Insert a Foley catheter. c. Start administering Pitocin. d. Immediately notify the care provider.

d. Immediately notify the care provider.

A new father states, "I know nothing about babies"; however, he seems to be interested in learning. How would the nurse best respond to this father? a. Continue to observe his interaction with the newborn. b. Tell him when he does something wrong. c. Show no concern; he will learn on his own. d. Include him in teaching sessions.

d. Include him in teaching sessions.

Because a full bladder prevents the uterus from contracting normally, nurses intervene to help the woman spontaneously empty her bladder as soon as possible. If all else fails, what tactic might the nurse use? a. Pouring water from a squeeze bottle over the woman's perineum b. Placing oil of peppermint in a bedpan under the woman c. Asking the healthcare provider to prescribe analgesic agents d. Inserting a sterile catheter

d. Inserting a sterile catheter

What is the most dangerous effect on the fetus of a mother who smokes cigarettes while pregnant? a. Genetic changes and anomalies b. Extensive CNS damage c. Fetal addiction to the substance inhaled d. Intrauterine growth restriction

d. Intrauterine growth restriction

Which statement related to the induction of labor is most accurate? a. Can be achieved by external and internal version techniques b. Is also known as a trial of labor (TOL) c. Is almost always performed for medical reasons d. Is rated for viability by a Bishop score

d. Is rated for viability by a Bishop score

Nafarelin is used to treat mild-to-severe endometriosis. What instruction or information should the nurse provide to a client regarding nafarelin administration? a. Nafarelin stimulates the secretion of gonadotropin-releasing hormone (GnRH), thereby stimulating ovarian activity. b. It should be administered by intramuscular (IM) injection. c. Nafarelin should be administered by a subcutaneous implant. d. It can cause the client to experience some hot flashes and vaginal dryness.

d. It can cause the client to experience some hot flashes and vaginal dryness.

The nurse is providing instruction to the newly delivered client regarding postbirth uterine and vaginal discharge, called lochia. Which statement is the most appropriate? a. Lochia is similar to a light menstrual period for the first 6 to 12 hours. b. It is usually greater after cesarean births. c. Lochia will usually decrease with ambulation and breastfeeding. d. It should smell like normal menstrual flow unless an infection is present.

d. It should smell like normal menstrual flow unless an infection is present.

Which fetal heart rate (FHR) finding is the most concerning to the nurse who is providing care to a laboring client? a. Accelerations with fetal movement b. Early decelerations c. Average FHR of 126 beats per minute d. Late decelerations

d. Late decelerations

The trend in the United States is for women to remain hospitalized no longer than 1 or 2 days after giving birth. Which scenario is not a contributor to this model of care? a. Wellness orientation model of care rather than a sick-care model b. Desire to reduce health care costs c. Consumer demand for fewer medical interventions and more family-focused experiences d. Less need for nursing time as a result of more medical and technologic advances and devices available at home that can provide information

d. Less need for nursing time as a result of more medical and technologic advances and devices available at home that can provide information

Which statement correctly describes the effects of various pain factors? a. Higher prostaglandin levels arising from dysmenorrhea can blunt the pain of childbirth. b. Upright positions in labor increase the pain factor because they cause greater fatigue. c. Women who move around trying different positions experience more pain. d. Levels of pain-mitigating beta-endorphins are higher during a spontaneous, natural childbirth.

d. Levels of pain-mitigating beta-endorphins are higher during a spontaneous, natural childbirth.

A woman gave birth to a healthy infant boy 5 days ago. What type of lochia does the nurse expect to find when evaluating this client? a. Lochia rubra b. Lochia sangra c. Lochia alba d. Lochia serosa

d. Lochia serosa

On examining a woman who gave birth 5 hours ago, the nurse finds that the woman has completely saturated a perineal pad within 15 minutes. What is the nurse's highest priority? a. Beginning an intravenous (IV) infusion of Ringer's lactate solution b. Assessing the woman's vital signs c. Calling the woman's primary health care provider d. Massaging the woman's fundus

d. Massaging the woman's fundus

Which alteration in cyclic bleeding best describes bleeding that occurs at any time other than menses? a. Oligomenorrhea b. Menorrhagia c. Leiomyoma d. Metrorrhagia

d. Metrorrhagia

A patient diagnosed with major depressive disorder refuses solid foods. In order to meet nutritional needs, which beverage will the nurse offer to this patient? a. Tomato juice b. Orange juice c. Hot tea d. Milk

d. Milk

An adolescent girl asks the school nurse for advice because she has dysmenorrhea. She says that a friend recommended she try an over-the-counter nonsteroidal anti-inflammatory drug (NSAID). The nurses response should be based on what? a. Hormone therapy is necessary for the treatment of dysmenorrhea. b. Acetaminophen is the drug of choice for the treatment of dysmenorrhea. c. Over-the-counter NSAIDs are rarely strong enough to provide adequate pain relief. d. NSAIDs are effective because they inhibit prostaglandins, leading to reduction in uterine activity.

d. NSAIDs are effective because they inhibit prostaglandins, leading to reduction in uterine activity.

A woman who is 16 weeks pregnant has come in for a follow-up visit with her partner. To reassure the client regarding fetal well-being, which is the highest priority action for the nurse to perform? a. Assess the fetal heart tones with a Doppler stethoscope. b. Measure and document the girth of the woman's abdomen. c. Complete an ultrasound examination (sonogram) in a timely manner. d. Offer the woman and her partner the opportunity to listen to the fetal heart tones.

d. Offer the woman and her partner the opportunity to listen to the fetal heart tones.

What strategy is considered one of the best for preventing smoking in teenagers? a. Large-scale printed information campaigns b. Emphasis on the long-term effects of smoking on health c. Threatening the social norms of groups most likely to smoke d. Peer-led programs emphasizing the social consequences of smoking

d. Peer-led programs emphasizing the social consequences of smoking

Which classification of placental separation is not recognized as an abnormal adherence pattern? a. Placenta accreta b. Placenta increta c. Placenta percreta d. Placenta abruptio

d. Placenta abruptio

Which basic type of pelvis includes the correct description and percentage of occurrence in women? a. Gynecoid: classic female pelvis; heart shaped; 75% b. Android: resembling the male pelvis; wide oval; 15% c. Anthropoid: resembling the pelvis of the ape; narrow; 10% d. Platypelloid: flattened, wide, and shallow pelvis; 3%

d. Platypelloid: flattened, wide, and shallow pelvis; 3%

An adolescent girl calls the nurse at the clinic because she had unprotected sex the night before and does not want to be pregnant. What should the nurse explain? a. It is too late to prevent an unwanted pregnancy. b. An abortion may be the best option if she is pregnant. c. The risk of pregnancy is minimal, so no action is necessary. d. Postcoital contraception is available to prevent implantation and therefore pregnancy.

d. Postcoital contraception is available to prevent implantation and therefore pregnancy.

Which assessment is least likely to be associated with a breech presentation? a. Meconium-stained amniotic fluid b. Fetal heart tones heard at or above the maternal umbilicus c. Preterm labor and birth d. Postterm gestation

d. Postterm gestation

When attempting to communicate with a client who speaks a different language, which action is the most appropriate? a. Promptly and positively respond to project authority. b. Never use a family member as an interpreter. c. Talk to the interpreter to avoid confusing the client. d. Provide as much privacy as possible.

d. Provide as much privacy as possible.

After giving birth to a healthy infant boy, a primiparous client, 16 years of age, is admitted to the postpartum unit. An appropriate nursing diagnosis for her is "Lack of understanding of infant care." What should the nurse be certain to include in the plan of care as he or she prepares the client for discharge? a. Teach the client how to feed and bathe her infant. b. Give the client written information on bathing her infant. c. Advise the client that all mothers instinctively know how to care for their infants. d. Provide time for the client to bathe her infant after she views a demonstration of infant bathing.

d. Provide time for the client to bathe her infant after she views a demonstration of infant bathing.

Many adolescents use alcohol for self-medication. How does an adolescent view the benefit of alcohol? a. Believes it has a stimulant effect b. Believes it increases alertness c. Provides a sense of euphoria d. Provides a defense against depression

d. Provides a defense against depression

Which term best describes the interval between the birth of the newborn and the return of the reproductive organs to their normal nonpregnant state? a. Involutionary period because of changes to the uterus b. Lochia period because of the nature of the vaginal discharge c. Mini-tri period because it lasts only 3 to 6 weeks d. Puerperium, or fourth trimester of pregnancy

d. Puerperium, or fourth trimester of pregnancy

As part of the discharge teaching, the nurse can prepare the mother for her upcoming adjustment to her new role by instructing her regarding self-care activities to help prevent postpartum depression (PPD). Which statement regarding this condition is most helpful for the client? a. Stay home and avoid outside activities to ensure adequate rest. b. Be certain that you are the only caregiver for your baby to facilitate infant attachment. c. Keep your feelings of sadness and adjustment to your new role to yourself. d. Realize that PPD is a common occurrence that affects many women.

d. Realize that PPD is a common occurrence that affects many women.

A 30-year-old multiparous woman has a boy who is years old and has recently delivered an infant girl. She tells the nurse, "I don't know how I'll ever manage both children when I get home." Which suggestion would assist this new mother in alleviating sibling rivalry? a. Tell the older child that he is a big boy now and should love his new sister. b. Let the older child stay with his grandparents for the first 6 weeks to allow him to adjust to the newborn. c. Ask friends and relatives not to bring gifts to the older sibling because you do not want to spoil him. d. Realize that the regression in habits and behaviors in the older child is a typical reaction and that he needs extra love and attention at this time.

d. Realize that the regression in habits and behaviors in the older child is a typical reaction and that he needs extra love and attention at this time.

A nurse is observing a family. The mother is holding the baby she delivered less than 24 hours ago. Her husband is watching his wife and asking questions about newborn care. The 4-year-old brother is punching his mother on the back. How should the nurse react to this situation? a. Report the incident to the social services department. b. Advise the parents that the toddler needs to be reprimanded. c. Report to oncoming staff that the mother is probably not a good disciplinarian. d. Realize that this is a normal family unit adjusting to a major family change.

d. Realize that this is a normal family unit adjusting to a major family change.

An 18-year-old pregnant woman, gravida 1, para 0, is admitted to the labor and birth unit with moderate contractions every 5 minutes that last 40 seconds. The client states, "My contractions are so strong, I don't know what to do." What should the nurse's first action be? a. Assess for fetal well-being. b. Encourage the woman to lie on her side. c. Disturb the woman as little as possible. d. Recognize that pain is personalized

d. Recognize that pain is personalized

What information should the nurse be aware of regarding telephonic nursing care such as warm lines? a. Were developed as a reaction to impersonal telephonic nursing care b. Were set up to take complaints concerning health maintenance organizations (HMOs) c. Are the second option when 9-1-1 hotlines are busy d. Refer to community service telephone lines designed to provide new parents with encouragement and basic information

d. Refer to community service telephone lines designed to provide new parents with encouragement and basic information

What are the most common causes for subinvolution of the uterus? a. Postpartum hemorrhage and infection b. Multiple gestation and postpartum hemorrhage c. Uterine tetany and overproduction of oxytocin d. Retained placental fragments and infection

d. Retained placental fragments and infection

When would an internal version be indicated to manipulate the fetus into a vertex position? a. Fetus from a breech to a cephalic presentation before labor begins b. Fetus from a transverse lie to a longitudinal lie before a cesarean birth c. Second twin from an oblique lie to a transverse lie before labor begins d. Second twin from a transverse lie to a breech presentation during a vaginal birth

d. Second twin from a transverse lie to a breech presentation during a vaginal birth

In evaluating the effectiveness of magnesium sulfate for the treatment of preterm labor, which finding alerts the nurse to possible side effects? a. Urine output of 160 ml in 4 hours b. DTRs 2+ and no clonus c. Respiratory rate (RR) of 16 breaths per minute d. Serum magnesium level of 10 mg/dl

d. Serum magnesium level of 10 mg/dl

In what form do families tend to be the most socially vulnerable? a. Married-blended family b. Extended family c. Nuclear family d. Single-parent family

d. Single-parent family

A patient with a diagnosis of depression and suicidal ideation was started on an antidepressant 1 month ago. When the patient comes to the community health clinic for a follow-up appointment he is cheerful and talkative. What priority assessment must the nurse consider for this patient? a. The medication dose needs to be decreased. b. Treatment is successful, and medication can be stopped. c. The patient is ready to return to work. d. Specific assessment for suicide plan must be evaluated.

d. Specific assessment for suicide plan must be evaluated.

A pregnant woman is at 38 weeks of gestation. She wants to know whether there are any signs that "labor is getting close to starting." Which finding is an indication that labor may begin soon? a. Weight gain of 1.5 to 2 kg (3 to 4 lb) b. Increase in fundal height c. Urinary retention d. Surge of energy

d. Surge of energy

A client has undergone a vaginal hysterectomy with a bilateral salpingo-oophorectomy. She is concerned about a loss of libido. What intervention by the nurse would be best? a. Suggest increasing vitamins and supplements daily. b. Discuss the value of a balanced diet and exercise. c. Reinforce that weight gain may be inevitable. d. Teach that estrogen cream inserted vaginally may help.

d. Teach that estrogen cream inserted vaginally may help.

What is one of the initial signs and symptoms of puerperal infection in the postpartum client? a. Fatigue continuing for longer than 1 week b. Pain with voiding c. Profuse vaginal lochia with ambulation d. Temperature of 38° C (100.4° F) or higher on 2 successive days

d. Temperature of 38° C (100.4° F) or higher on 2 successive days

A woman at 26 weeks of gestation is being assessed to determine whether she is experiencing preterm labor. Which finding indicates that preterm labor is occurring? a. Estriol is not found in maternal saliva. b. Irregular, mild uterine contractions are occurring every 12 to 15 minutes. c. Fetal fibronectin is present in vaginal secretions. d. The cervix is effacing and dilated to 2 cm.

d. The cervix is effacing and dilated to 2 cm.

Which explanation concerning postpartum ovary function is most accurate? a. Almost 75% of women who do not breastfeed resume menstruating within 1 month after birth. b. Ovulation occurs slightly earlier for breastfeeding women. c. Because of menstruation and ovulation schedules, contraception considerations can be postponed until after the puerperium. d. The first menstrual flow after childbirth usually is heavier than normal.

d. The first menstrual flow after childbirth usually is heavier than normal.

In follow-up appointments or visits with parents and their new baby, it is useful if the nurse can identify infant behaviors that can either facilitate or inhibit attachment. What is an inhibiting behavior? a. The infant cries only when hungry or wet. b. The infant's activity is somewhat predictable. c. The infant clings to the parents. d. The infant seeks attention from any adult in the room.

d. The infant seeks attention from any adult in the room.

A Filipino American patient had a nursing diagnosis of situational low self-esteem related to poor social skills as evidenced by lack of eye contact. Interventions were applied to increase the patient's self-esteem but after 3 weeks, the patient's eye contact did not improve. What is the most accurate analysis of this scenario? a. The patient's eye contact should have been directly addressed by role playing to increase comfort with eye contact. b. The nurse should not have independently embarked on assessment, diagnosis, and planning for this patient. c. The patient's poor eye contact is indicative of anger and hostility that were unaddressed. d. The nurse should have assessed the patient's culture before making this diagnosis and plan.

d. The nurse should have assessed the patient's culture before making this diagnosis and plan.

What best describes central nervous system (CNS) stimulants? a. Acute intoxication can lead to coma. b. They produce strong physical dependence. c. Withdrawal symptoms are life threatening. d. They can result in strong psychologic dependence.

d. They can result in strong psychologic dependence.

In which situation would the nurse be called on to stimulate the fetal scalp? a. As part of fetal scalp blood sampling b. In response to tocolysis c. In preparation for fetal oxygen saturation monitoring d. To elicit an acceleration in the fetal heart rate (FHR)

d. To elicit an acceleration in the fetal heart rate (FHR)

During a well-child visit, the nurse plots the childs BMI on the health record. What is the purpose of the BMI? a. To determine medication dosages b. To predict adult height and weight c. To identify coping strategies used by the child d. To provide a consistent measure of obesity

d. To provide a consistent measure of obesity

Nurses need to understand the basic definitions and incidence data regarding post-partum hemorrhage (PPH). Which statement regarding this condition is most accurate? a. PPH is easy to recognize early; after all, the woman is bleeding. b. Traditionally, it takes more than 1000 ml of blood after vaginal birth and 2500 ml after cesarean birth to define the condition as PPH. c. If anything, nurses and physicians tend to overestimate the amount of blood loss. d. Traditionally, PPH has been classified as early PPH or late PPH with respect to birth.

d. Traditionally, PPH has been classified as early PPH or late PPH with respect to birth.

The nurse suspects that her postpartum client is experiencing hemorrhagic shock. Which observation indicates or would confirm this diagnosis? a. Absence of cyanosis in the buccal mucosa b. Cool, dry skin c. Calm mental status d. Urinary output of at least 30 ml/hr

d. Urinary output of at least 30 ml/hr

A patient diagnosed with major depressive disorder is receiving imipramine 200 mg qhs. Which assessment finding would prompt the nurse to collaborate with the health care provider regarding potentially hazardous side effects of this drug? a. Dry mouth b. Blurred vision c. Nasal congestion d. Urinary retention

d. Urinary retention

When a nurse is counseling a woman experiencing primary dysmenorrhea, which nonpharmacologic intervention might be recommended? a. Increasing the intake of red meat to replace blood loss b. Reducing the intake of diuretic foods, such as peaches and asparagus c. Temporarily substituting physical activity for a sedentary lifestyle d. Using a heating pad on the abdomen to relieve cramping

d. Using a heating pad on the abdomen to relieve cramping

Which sign or symptom is considered a first-trimester warning sign and should be immediately reported by the pregnant woman to her health care provider? a. Nausea with occasional vomiting b. Fatigue c. Urinary frequency d. Vaginal bleeding

d. Vaginal bleeding

Which action would the nurse teach to help the client prevent vulvovaginitis? a. Wipe back to front after urination. b. Cleanse the inner labial mucosa with soap and water. c. Use feminine hygiene sprays to avoid odor. d. Wear loose cotton underwear.

d. Wear loose cotton underwear.

Many pregnant women have questions regarding work and travel during pregnancy. Which education is a priority for the nurse to provide? a. Women should sit for as long as possible and cross their legs at the knees from time to time for exercise. b. Women should avoid seat belts and shoulder restraints in the car because they press on the fetus. c. Metal detectors at airport security checkpoints can harm the fetus if the woman passes through them several times. d. While working or traveling in a car or on an airplane, women should arrange to walk around at least every hour or so.

d. While working or traveling in a car or on an airplane, women should arrange to walk around at least every hour or so.

A woman gave birth vaginally to a 9-pound, 12-ounce girl yesterday. Her primary health care provider has written orders for perineal ice packs, use of a sitz bath three times daily, and a stool softener. Which information regarding the client's condition is most closely correlated with these orders? a. Woman is a gravida 2, para 2. b. Woman had a vacuum-assisted birth. c. Woman received epidural anesthesia. d. Woman has an episiotomy.

d. Woman has an episiotomy.

A Puerto Rican American patient uses dramatic body language when describing emotional discomfort. Which analysis most likely explains the patient's behavior? The patient a. has a histrionic personality disorder. b. believes dramatic body language is sexually appealing. c. wishes to impress staff with the degree of emotional pain. d. belongs to a culture in which dramatic body language is the norm.

d. belongs to a culture in which dramatic body language is the norm.

A nurse provided medication education for a patient diagnosed with major depressive disorder who began a new prescription for phenelzine (Nardil). Which behavior indicates effective learning? The patient a. monitors sodium intake and weight daily. b. wears support stockings and elevates the legs when sitting. c. can identify foods with high selenium content that should be avoided. d. confers with a pharmacist when selecting over-the-counter medications.

d. confers with a pharmacist when selecting over-the-counter medications.

A disheveled patient in the acute phase of major depressive disorder is withdrawn, has psychomotor retardation, and has not showered for several days. The nurse will a. bring up the issue at the community meeting. b. calmly tell the patient, "You must bathe daily." c. make observations about the patient's poor personal hygiene. d. firmly and neutrally assist the patient with showering.

d. firmly and neutrally assist the patient with showering.

A nurse worked with a patient diagnosed with major depressive disorder, severe withdrawal, and psychomotor retardation. After 3 weeks, the patient did not improve. The nurse is most at risk for feelings of a. guilt and despair. b. over-involvement. c. interest and pleasure. d. ineffectiveness and frustration.

d. ineffectiveness and frustration.


Ensembles d'études connexes

Real Estate, Chapter 2, Section 3 (Leasehold Estates), Pop Quiz Questions 3.1

View Set

Multinational Finance: Financial Goals and Corporate Governance

View Set

Chapter 9: Embracing Diversity and Inclusions

View Set

Latitude & Longitude (fill in the blank)

View Set